Как найти давление газа при увеличении объема

Содержание:

  1. Свойства газов
  2. Давление газов
  3. Зависимость между объёмом и давлением газа. Закон Еойля — Мариотта
  4. Зависимость между плотностью газа и его давлением
  5. Зависимость объёма газа от температуры. Закон Гей-Люссака
  6. Зависимость давления газа от температуры. Закон Шарля
  7. Абсолютная шкала температур
  8. Зависимость между объёмом, давлением и температурой газа
  9. Физическая сущность понятия абсолютного нуля
  10. Изменение температуры газа при быстром расширении и сжатии
  11. Применение сжатых газов

Газ — это одно из трёх основных агрегатных состояний вещества, характеризующееся очень слабыми связями между составляющими его частицами (молекулами, атомами или ионами), а также их большой подвижностью. Частицы газа почти свободно и хаотически движутся в промежутках между столкновениями, во время которых происходит резкое изменение характера их движения.

На странице -> решение задач по физике собраны решения задач и заданий с решёнными примерами по всем темам физики.

Свойства газов

Главные свойства газов – это подвижность и хаотичное движение частиц, направление которых меняется при столкновении. Газ – одно из 4 агрегатных состояний веществ, которые на сегодняшний день известны науке.

Давление газов

Всякий газ производит давление на оболочку, внутри которой он находится.

Давление, производимое газом на стенки сосуда, объясняется ударами движущихся молекул.

При ударе о стенку молекулы газа отдают ей определённое количество движения; стенка испытывает при этом действие некоторой силы.

Удар каждой отдельной молекулы о стенку сосуда производит очень небольшое действие. Но молекул газа очень много, удары о стенки сосуда происходят беспрерывно, поэтому в результате получается значительное давление.

Хаотичность движения молекул приводит к тому, что давление газа одинаково во всех направлениях.

При нагревании давление газа увеличивается. Так как при этом число молекул газа не изменяется, то увеличение давления можно объяснить только тем, что удары молекул о стенки заключающего газ сосуда делаются при нагревании чаще и что каждый удар становится сильнее. Удары же могут стать чаще и сильнее, если увеличивается скорость движения молекул. Это подтверждается, как мы видели (гл. V), многочисленными опытами.

Зависимость между объёмом и давлением газа. Закон Еойля — Мариотта

Состояние газа определяется его объёмом, давлением и температурой. С изменением этих величин меняется и состояние газа. Мы будем рассматривать последовательно процессы, при которых одна из величин, характеризующих состояние газа, постоянна, а две другие меняются.

Изучим сначала такой процесс, при котором давление и объём газа изменяются, а температура остаётся постоянной. Такой процесс называется изотермическим 1.

1 От греч. слов: изос — равный, термос — тёплый.

Итак, рассмотрим, как изменяется давление данной массы газа при изменении его объёма, если температура газа не меняется.

Опыты, устанавливающие эту зависимость, можно произвести на приборе, три положения которого изображены на рисунке 134.

Свойства газов в физике

Рис. 134. Прибор для установления зависимости между объёмом и давлением газа (в трёх положениях).

В этом приборе стеклянная трубка А соединяется резиновой трубкой с другой стеклянной трубкой В. Трубка А вверху снабжена краном Свойства газов в физике обе трубки наполняются ртутью.

Откроем кран Свойства газов в физике и установим трубку В так, чтобы уровень ртути в трубке А был, например, на середине трубки (положение I). Давление над ртутью в обеих трубках атмосферное; допустим, что оно равно 76 см рт. ст. Закроем теперь кран, отделив этим массу воздуха в трубке А от атмосферного воздуха. Таким образом, в этой стадии опыта мы будем иметь в трубке А определённую массу воздуха, находящегося под давлением p1 = 76 см рт. ст.

Поднимем теперь трубку В вверх на столько, чтобы объём воздуха в трубке А уменьшился вдвое (положение II). Уровень ртути в трубке В при этом значительно поднимется над уровнем в трубке А.

Рассмотрим теперь, чему будет равно давление воздуха в трубке А. Это давление уравновешивает атмосферное давление и давление всего столба ртути в трубке В, стоящего выше уровня n1 высота этого столба n1n оказывается равной 76 см. Таким образом, давление воздуха в трубке А уравновешивает не одну, как в первом случае, а две атмосферы 2 = 2 am).

Значит, с уменьшением объёма данной массы газа в два раза давление его увеличивается в два раза. Если уменьшить объём газа в 1,5; 2,5; 3 раза, то соответственно в 1,5; 2,5; 3 раза увеличится его давление.

Опустим теперь трубку В так, чтобы масса воздуха в трубке А заняла вдвое больший объём (положение III). Уровень ртути в трубке А при этом понизится. Атмосферное давление теперь уравновешивает давление воздуха в трубке А и давление столба ртути от уровня n в трубке А до уровня в трубке В. Измерения показывают, что высота этого столба ртути равна 38 см. Давление, производимое воздухом в трубке А, найдём, вычтя из атмосферного давления давление столба ртути: р3 = 76 см—38 см = 38 см; следовательно, р3 = 0,5 am.

Итак, при увеличении объёма газа в два раза его давление уменьшается в два раза.

Перемещая трубку В в различные положения и отсчитывая каждый раз объём и давление воздуха в трубке А, найдём, что при уменьшении объёма исследуемой массы воздуха в некоторое число раз давление его увеличивается во столько же раз. Температура воздуха при всех опытах остаётся постоянной.

Опыты, проведённые с другими газами, дали те же результаты.

Изучая на опыте зависимость давления газа от его объёма, английский учёный Бойль (1627—1691) и французский учёный Мариотт (1620—1684) независимо один от другого открыли следующий закон.

Давление данной массы газа при неизменной температуре обратно пропорционально объёму газа.

Этот закон называется законом Бойля — Мариотта.

Выразим закон Бойля — Мариотта математически. Пусть температура некоторой массы газа постоянная и пусть:

V1 — объём газа при давлении р1,

V2  » » » » р2.

Согласно закону Бойля — Мариотта можно написать:

Свойства газов в физике

Из этой формулы следует, что:

Свойства газов в физике

Полученное равенство можно рассматривать как новое выражение закона Бойля — Мариотта.

Произведение объёма данной массы газа на его давление при неизменной температуре есть величина постоянная.

Изобразим графически изотермическое изменение состояния газа. Для этого по оси абсцисс будем откладывать значения объёмов газа, а по оси ординат соответствующие им значения давлений. Выберем масштаб так, чтобы начальные значения объёма и давления были равны 1. Тогда начальное состояние газа будет изображено точкой А (рис. 135). Если давление увеличится вдвое, объём уменьшится в два раза, состояние газа изобразится на графике точкой В. При уменьшении первоначального давления вдвое объём удвоится, получим точку С. Беря далее давления в три, четыре и т. д. раза больше или меньше начального, а объёмы соответственно в три, четыре и т. д. раза меньше или больше, получим ряд точек, изображающих различные состояния одной и той же массы газа при одинаковой температуре.

Свойства газов в физике

Рис. 135. График изотермического процесса.

Проведя через эти точки линию, получим кривую, которая называется изотермой.

Тщательными исследованиями установлено, что для реально существующих газов закон Бойля — Мариотта имеет лишь приближённое значение. Так, например, если произведение pV при 1 am равно единице, то при 2 am оно имеет следующие значения:

для воздуха ………………………0,99977

» водорода …………………….. 1,00026

» окиси углерода………………….. 0,99974

» двуокиси углерода………………… 0,99720

При очень больших давлениях (в сотни и тысячи атмосфер) закон Бойля — Мариотта становится совершенно неприменимым; в таких случаях зависимость между объёмом и давлением газа выражается более сложными уравнениями.

Зависимость между плотностью газа и его давлением

Плотность газа численно равна массе, заключённой в единице объёма.

Масса газа не меняется при его сжатии или расширении, но объём меняется; следовательно, меняется и плотность газа.

Пусть при постоянной температуре: D1 — плотность газа при объёме V1 и давлении p1 a D2 —  плотность газа при объёме V2 и давлении p2.

Если масса газа равна m, то можно написать:

Свойства газов в физике

откуда:

Свойства газов в физике

Но Свойства газов в физике на основании закона Бойля — Мариотта; поэтому

Свойства газов в физике

При постоянной температуре плотность газа прямо пропорциональна его давлению.

Нетрудно понять справедливость этого вывода, исходя из молекулярно-кинетической теории. В самом деле, давление газа обусловлено ударами его молекул. Если объём газа уменьшится вдвое, то в новом объёме , плотность газа станет вдвое больше. Вдвое увеличится и число ударов молекул о стенки, т. е. давление газа возрастёт в два раза.

Зависимость объёма газа от температуры. Закон Гей-Люссака

Как и все тела, газы при нагревании расширяются, причём весьма заметно даже при незначительном нагревании. Это легко обнаружить на следующем простом опыте (рис. 136).

Свойства газов в физике

Рис. 136. Установка для наблюдения расширения газа при нагревании.

Колба А соединяется с расположенной горизонтально трубкой CD, которая укреплена вдоль шкалы. Внутри этой трубки находится небольшой столбик ртути. Достаточно к колбе прикоснуться рукой, как столбик ртути в трубке CD начнёт двигаться.

При охлаждении колбы столбик ртути перемещается влево, а при нагревании — вправо; следовательно, газ при охлаждении сжимается, а при нагревании расширяется. Зная объём колбы и диаметр трубки, можно измерить увеличение объёма газа.

Постепенно нагревая газ в колбе, можно установить, что при постоянном давлении изменение объёма данной массы газа пропорционально изменению температуры. Поэтому тепловое расширение газа, так же как и других тел, можно охарактеризовать при помощи коэффициента объёмного расширения.

Пусть при температуре 0°С объём газа равен V0 , а при температуре t объём Vt. Увеличение объёма, приходящееся на каждую единицу объёма, взятого при 0°С, при нагревании на один градус будет равно:

Свойства газов в физике

откуда: Свойства газов в физике (1)

Величина Свойства газов в физике входящая в писанные выше формулы, называется коэффициентом объёмного расширения газа.

Свойства газов в физике

Жозеф Луи Гей-Люссак (1778—1850)— один из выдающихся французских химиков и физиков. Он открыл ряд важных химических и физических законов, из которых в физике широко известен закон одинакового расширения газов и паров при одинаковом повышении температуры.

Французский учёный Гей-Люссак, исследуя на опыте тепловое расширение газов, открыл, что, коэффициент объёмного расширения у всех газов при постоянном давлении одинаков и численно равен Свойства газов в физике

В этом отношении расширение газов при нагревании отличается от расширения твёрдых и жидких тел, где, как мы видели (см. § 81 и 82), коэффициент объёмного расширения зависит от химического состава тел.

Положим в формуле (1): Свойства газов в физике

получим: Свойства газов в физике откуда следует, что при нагревании на 1° под постоянным давлением объём данной массы газа увеличивается на Свойства газов в физике того объёма, который газ занимал при 0°С.

Этот закон получил название закона Гей-Люссака. Процессы, подобные рассмотренному, протекающие при постоянном давлении, называются изобарными1.

1 От греч. слов: изос — равный, барос — тяжесть, вес.

Формула (1) показывает, что объём газа при температуре равен произведению его объёма, взятого при 0°С, на двучлен объёмного расширения Свойства газов в физике

Пример. 1. Объём некоторой массы газа при 0°С равен 10 л. Найти объём его при t=273°С, если давление постоянно.

По условиям задачи нам известен объём газа при 0°С, т. е. V0 = 10 л; подставляя числовые данные задачи в формулу Свойства газов в физике найдем, что

Свойства газов в физике

Пример 2. При температуре 273°С объём некоторой массы газа равен 10 л. Чему будет равняться объём этого газа при температуре 546°С, если давление постоянно?

Нам известен объём газа при температуре 273°С; чтобы определить объём этого газа при t2 = 546°С, надо предварительно найти его объём при 0°.

Этот объём найдётся из равенства:

Свойства газов в физике

откуда:

Свойства газов в физике

Найдём теперь объём газа при 546°:

Свойства газов в физике

Зависимость давления газа от температуры. Закон Шарля

Нагревая газ в закрытом цилиндре, например в папиновом котле (рис. 136а), можно по манометру заметить, что давление газа увеличивается. Следя по термометру за повышением температуры, легко установить, что при постоянном объёме давление газа возрастает пропорционально повышению температуры.

Свойства газов в физике

Рис. 136а. При нагревании газа в закрытом цилиндре давление его повышается.

Аналогично тому, как для характеристики теплового расширения газов мы ввели коэффициент объёмного расширения, введём величину, характеризующую изменение давления газа при изменении его температуры.

Обозначим буквой р0 давление газа при 0°С, a pt — давление при. Увеличение давления, приходящееся на каждую единицу начального давления при нагревании на 1°С, будет равно:

Свойства газов в физике  (1)

Величина Свойства газов в физике (греч. «гамма») называется термическим коэффициентом давления газа.

Измерения показывают, что величина термического коэффициента давления для всех

газов одинакова и равна Свойства газов в физике

Определяя из формулы (1) величину pt получим:

Свойства газов в физике   (2)

Положим в формуле (2) Свойства газов в физикетогда Свойства газов в физике

Отсюда следует, что давление данной массы газа при нагревании на 1° при постоянном объеме увеличивается на Свойства газов в физике того давления, которым обладал газ при 0°C. 

Этот закон называется законом Шарля, по имени французского учёного, открывшего его в 1787 г.

Из закона Шарля следует, что термический коэффициент давления газа Свойства газов в физике равен коэффициенту объёмного расширения Свойства газов в физике Это равенство вытекает из закона Бойля — Мариотта. Докажем это.

Пусть некоторая масса газа заключена в цилиндре под поршнем (рис. 137, а) и пусть температура её в этом начальном состоянии равна 0°, объём V0 и давление р0. Закрепим поршень АВ и нагреем газ до температуры (рис. 137, б); тогда давление газа увеличится и станет равным рt объём же его останется прежним.

По закону Шарля: Свойства газов в физике

Будем теперь газ нагревать от 0 до (рис. 137, в), предоставив поршню свободно перемещаться. Давление газа останется таким же, каким было в начальном его состоянии, т. е. р, объём же увеличится до Vt. По закону Гей-Люссака:

Свойства газов в физике

Свойства газов в физике

Рис. 137.

а)    начальное состояние газа: 0°, V0, р0;

б)    состояние газа, определяемое величинами: Свойства газов в физике

в)    состояние газа, определяемое величинами: Свойства газов в физике

Итак, имеем: при температуреобъём данной массы газа V0 и давление Свойства газов в физике при той же температуре: давление ри объём Свойства газов в физике По закону Бойля— Мариотта:

Свойства газов в физике

После упрощения этого выражения получаем равенство:

Свойства газов в физике

Выразим сначала в виде таблицы, а потом графически зависимость давления газа от температуры. Для этого воспользуемся уравнением:

Свойства газов в физике

Свойства газов в физике

Рис. 138. График изменения давления газа от температуры.

Отложим по оси абсцисс в некотором условном масштабе температуры газа, а по оси ординат соответствующие этим температурам давления, взятые из написанной выше таблицы.

Соединяя на графике отмеченные точки, получим прямую LM (рис. 138), представляющую собой график зависимости давления газа от температуры при постоянном объёме.

Процесс изменения состояния газа, происходящий при неизменном объёме газа, называется  изохорным 1 процессом, а линия LM, изображающая изменение давления газа при постоянном объеме в зависимости от температуры, называется изохорой.

1 От греч. слов: изос — разный, хорема — вместимость.

Пример 1. Давление газа при 0°С равно 780 мм рт. ст. Определить давление этого газа при температуре 273°С.

По формуле Свойства газов в физике найдем, что

Свойства газов в физике

Пример 2. Чему будет равно давление газа при температуре 546°, если давление его при температуре 273° равно 780 мм рт. cm.?

В этой задаче прежде всего надо определить давление газа при 0°С. По формуле Свойства газов в физике находим:

Свойства газов в физике

Теперь можно определить давление газа при t = 546°:

Свойства газов в физике

Законы Гей-Люссака и Шарля так же, как и закон Бойля — Мариотта, лишь приближённо отражают свойства газов. Это можно видеть хотя бы. из того факта, что для разных газов величины Свойства газов в физике и Свойства газов в физике несколько различаются между собой (см. таблицу).

Свойства газов в физике      

Точные измерения показывают, что для каждого данного газа значения Свойства газов в физикеи Свойства газов в физикеполучаются разные в зависимости от того, в каком температурном интервале и при каком давлении они определены. Однако эти различия очень незначительны, они учитываются лишь при весьма точных расчётах.

Абсолютная шкала температур

Вернёмся ещё раз к графику изменения давления газа с температурой (рис. 138).

Продолжим прямую LM на этом графике до пересечения её с горизонтальной осью, по которой откладываются температуры газа, она пересечёт эту ось в точке K. Отрезок ОК будет изображать на этом графике такую температуру газа, при которой давление его равно нулю. Чему равна эта температура?

Обратимся к уравнению Свойства газов в физике Положим в этом уравнении pt = 0, т. е. напишем следующее равенство:

Свойства газов в физике

Так как давление газа при 0°С не равно нулю Свойства газов в физике то из написанного равенства следует, что:

Свойства газов в физике

откуда: Свойства газов в физике или, так как

Свойства газов в физике

Итак, давление газа равняется нулю при температуре —273°С.

Свойства газов в физике

Вильям Томсон (Кельвин) (1824— 1907) — выдающийся английский физик. Ему принадлежат важные открытия в области теории электричества и теплоты и изобретения, из которых наиболее значительным было усовершенствование телеграфной связи. Он ввёл в физику понятие об абсолютной температуре. Его именем названы градусы шкалы абсолютных температур — градусы Кельвина.

Английский учёный Вильям Томсон (Кельвин) предложил такую шкалу температур, при которой за нуль градусов принята температура — 273°. Эта шкала получила название абсолютной шкалы температур, или шкалы Кельвина, а нуль градусов этой шкалы, равный — 273°, называется абсолютным нулём температур.

В шкале Кельвина величина градуса та же, что и в стоградусной шкале.

Будем обозначать температуру по шкале Кельвина буквой Т.

При нормальном атмосферном давлении температура таяния льда по шкале Кельвина Т0 = 273°, температура же кипения воды = 373°.

Всякая другая температура стоградусной шкалы связана с абсолютной температурой Т соотношениями:

Свойства газов в физике

Зависимость между объёмом, давлением и температурой газа

Объединённый закон газового состояния. Мы рассмотрели процессы, в которых одна из трёх величин, характеризующих состояние газа (объём, давление и температура), не меняется.

Вы видели, что если не меняется температура, то давление и объём газа связаны друг с другом законом Бойля —- Мариотта. При постоянном давлении объём газа изменяется с изменением температуры по закону Гей-Люссака, и, наконец, при постоянном объёме давление газа меняется с изменением температуры по закону Шарля.

Однако в природе часто имеют место процессы, когда одновременно меняются все три величины, характеризующие состояние газа. Установим теперь, какая связь существует между объёмом, давлением и температурой.

Пусть для двух каких-либо произвольных состояний некоторой массы газа эти величины будут:

Свойства газов в физике

Из этих состояний изменением величин р, V или t газ можно перевести в любые другие состояния. Будем, например, сохраняя постоянным давление, переводить газ из состояний 1) и 2) в состояния, при которых температура газа будет равна 0°С.

По закону Гей-Люссака объём газа V1 после уменьшения температуры от до 0° будет равен Свойства газов в физике объём V2, после уменьшения температуры от t2 до 0° будет Свойства газов в физике

Новые состояния газа выразятся так:

Свойства газов в физике

В обоих этих состояниях температура газа одинакова, поэтому на основании закона Бойля — Мариотта можно написать:

Свойства газов в физике  (1)

Так как величины р, V, t, характеризующие состояние рассматриваемого газа и обозначенные индексами 1 и 2, выбраны были нами произвольно, то равенство (1) справедливо для любых состояний этого газа. Поэтому можно утверждать, что:

Свойства газов в физике  (2)

Для данной массы газа произведение давления газа на его объём, делённое на двучлен объёмного расширения, есть величина постоянная.

Выведенная нами зависимость между объёмом, давлением и температурой газа называется объединённым законом газового состояния, а равенство (1) или (2) — уравнением состояния газа.

Уравнение состояния газа можно упростить, введя в него вместо температуры t по стоградусной шкале температуру Т по абсолютной шкале температур. Для этого преобразуем уравнение:

Свойства газов в физике

Введя в него значение Свойства газов в физике получим:

Свойства газов в физике

что после сокращения на 273 даст:

Свойства газов в физике

Но Свойства газов в физике и Свойства газов в физике; следовательно, можно написать:

Свойства газов в физике

Это означает, что для данной массы газа произведение давления на объём, делённое на абсолютную температуру, постоянно при всех температурах: Свойства газов в физике

В частности, если при температуре Т = 273° объём газа равен V0 и давление его р, то можно написать:

Свойства газов в физике

Физическая сущность понятия абсолютного нуля

Мы уже отмечали, что реальные газы лишь приближённо следуют законам Гей-Люссака, Шарля и Бойля — Мариотта. Однако можно представить себе газ, для которого эти законы выполнялись бы в точности. Молекулы такого газа можно представить себе в виде упругих шариков исчезающе малого объёма, взаимодействие между которыми осуществляется только через их столкновения друг с другом. В физике такой газ принято называть идеальным газом.

Из уравнения Свойства газов в физике следует, что при t = —273°, т. е. при абсолютном нуле, давление газа равно нулю. Но ведь давление газа есть результат ударов движущихся молекул о стенки сосуда. Следовательно, при температуре абсолютного нуля должно прекратиться тепловое движение молекул идеального газа.

Опыт показывает, что при малых давлениях свойства реальных газов очень близки к свойствам идеального газа. Следовательно, при приближении к температуре абсолютного нуля должно прекратиться тепловое движение молекул и реального газа. Этот вывод относится не только к газам, но и к твёрдым и жидким телам.

Физикой установлено, что такое состояние вещества недостижимо, но к нему можно подойти очень близко. В настоящее время достигнута температура, которая выше абсолютного нуля всего на несколько стотысячных долей градуса.

Изменение температуры газа при быстром расширении и сжатии

Опыты показывают, что при быстром сжатии температура газа повышается, а при быстром расширении понижается.

Увеличение температуры газа при сжатии можно показать на следующем простом опыте. Возьмём толстостенный цилиндрический стеклянный сосуд, внутри которого может двигаться поршень (рис. 139). При быстром сжатии воздух в сосуде сильно нагревается, и легко воспламеняющееся вещество (например, ватка, смоченная эфиром), положенное на дно сосуда, вспыхивает. Такого рода явление используется, например, в двигателях внутреннего сгорания —дизелях: при сжатии воздуха в цилиндре двигателя горючая смесь, введённая в цилиндр, нагревается до температуры воспламенения (работа двигателя описана в § 131).

Свойства газов в физике

Рис. 139. При быстром сжатии воздух в цилиндре сильно нагревается и легко воспламеняющееся вещество вспыхивает.

При быстром же расширении газа температура его понижается. Это можно наблюдать на следующем опыте. Будем накачивать воздух в прочную закрытую пробкой стеклянную банку, содержащую пары воды. При достижении определённого давления пробка выскочит; при этом воздух, расширяясь, совершит работу и охладится, вследствие чего водяной пар превратится в туман (рис. 140).

Свойства газов в физике

Рис. 140. Сжатый в сосуде воздух, выбрасывая пробку, расширяется. Совершая при этом работу, он охлаждается, вследствие чего водяной пар в сосуде превращается в туман.

Понижение температуры при быстром расширении газа используется для получения сжиженных газов; об этом будет рассказано в § 122.

Изменение температуры тела, как было установлено в § 71, связано с изменением внутренней энергии тела. Так как при быстром сжатии температура газа повышается, то внутренняя энергия его при этом увеличивается. Увеличение внутренней энергии газа происходит в результате работы, совершённой при его сжатии. Расширяясь же, газ совершает работу; при этом внутренняя энергия его уменьшается, и если расширение происходит быстро, то температура газа, как мы видели в наших опытах, понижается.

Процесс, происходящий в теле без теплообмена с окружающими его другими телами, называется адиабатным процессом.

Все быстро протекающие процессы практически могут считаться адиабатными.

Применение сжатых газов

Многие сжатые газы в настоящее время находят широкое применение в технике.

Сжатый воздух, например, применяется в работе различных пневматических инструментов: отбойных молотков, заклёпочных молотков, в разбрызгивателях краски и др.

На рисунке 141 показана схема устройства отбойного молотка. Сжатый воздух подаётся в молоток по шлангу М. Золотники Z, аналогичные применяемым в паровых машинах, направляют его поочерёдно то в заднюю, то в переднюю часть цилиндра. Поэтому воздух давит на поршень Р то с одной, то с другой стороны, что вызывает быстрое возвратно-поступательное движение поршня и пики молотка В. Последняя наносит быстро следующие друг за другом удары, внедряется в уголь и откалывает куски его от массива.

Свойства газов в физике

Рис. 141. Схема устройства отбойного молотка.

Существуют также пескоструйные аппараты, которые дают сильную струю воздуха, смешанную с песком. Эти аппараты применяются, например, для очистки стен. Сейчас нередко можно видеть работу специальных аппаратов, применяемых для окраски стен, где краска распыляется сжатым воздухом. Сжатым воздухом открываются двери вагонов метро и троллейбусов. Сжатый воздух используется в работе тормозов на транспорте. Схематическое устройство одного из видов пневматического тормоза железнодорожного вагона изображено на рисунке 142.

Компрессор подаёт воздух по магистрали в стальной резервуар А. Поршень В тормозного цилиндра оказывается под одинаковым давлением справа и слева; поэтому соединённая с ним тормозная колодка D отжата от колеса. Если открыть тормозной кран М, то находящийся в магистрали под давлением воздух устремится в атмосферу; клапан К захлопнется, и, таким образом, стальной резервуар изолируется от магистрали. Теперь давление на поршень В справа станет больше, чем давление слева, вследствие чего тормозная колодка прижмётся к ободу колеса. Если теперь кран М закрыть и снова подать в магистраль сжатый воздух, то восстановится первоначальное положение.

Свойства газов в физике

Рис. 142. Схема устройства железнодорожного пневматического тормоза.

В технике применяется не только сжатый воздух, но и некоторые другие газы, так, например, водород, ацетилен и кислород применяются при газовой сварке; аммиак используется в холодильном деле. Чтобы газы было удобно перевозить, их помещают в прочные стальные баллоны, накачивая до давления 60—200 am.

Свойства газов в физике

Рис. 142а. Внешний вид мощного компрессора.

Сжатие газов осуществляется с помощью мощных нагнетательных насосов — компрессоров.

На рисунке 143, а, б дана схема работы компрессора.

Компрессор состоит из цилиндра с поршнем и двумя клапанами; один из них входной, другой выходной. При движении поршня вниз (рис. 143, б) открывается входной клапан и в цилиндр поступает воздух из помещения; при движении поршня вверх (рис. 143, а) входной клапан закрывается, вошедший воздух сжимается поршнем и через выходной клапан поступает в стальной баллон для хранения сжатого газа.

Существуют так называемые многоступенчатые компрессоры, в которых газ последовательно  

сжимается в трёх или четырёх цилиндрах. Такие компрессоры позволяют получить газ, сжатый до давления в тысячи атмосфер. На рисунке 142а изображён внешний вид одного из типов многоступенчатых компрессоров.

Свойства газов в физике

Рис. 143, а, б. Схема работы компрессора.

Услуги по физике:

  1. Заказать физику
  2. Заказать контрольную работу по физике
  3. Помощь по физике

Лекции по физике:

  1. Физические величины и их измерение
  2. Основные законы механики
  3. Прямолинейное равномерное движение
  4. Прямолинейное равнопеременное движение
  5. Сила
  6. Масса
  7. Взаимодействия тел
  8. Механическая энергия
  9. Импульс
  10. Вращение твердого тела
  11. Криволинейное движение тел
  12. Колебания
  13. Колебания и волны
  14. Механические колебания и волны
  15. Бегущая волна
  16. Стоячие волны
  17. Акустика
  18. Звук
  19. Звук и ультразвук
  20. Движение жидкости и газа
  21. Молекулярно-кинетическая теория
  22. Молекулярно-кинетическая теория строения вещества
  23. Молекулярно — кинетическая теория газообразного состояния вещества
  24. Теплота и работа
  25. Температура и теплота
  26. Термодинамические процессы
  27. Идеальный газ
  28. Уравнение состояния идеального газа
  29. Изменение внутренней энергии
  30. Переход вещества из жидкого состояния в газообразное и обратно
  31. Кипение, свойства паров, критическое состояние вещества
  32. Водяной пар в атмосфере
  33. Плавление и кристаллизация
  34. Тепловое расширение тел
  35. Энтропия
  36. Процессы перехода из одного агрегатного состояния в другое
  37. Тепловое расширение твердых и жидких тел
  38. Свойства жидкостей
  39. Свойства твёрдых тел
  40. Изменение агрегатного состояния вещества
  41. Тепловые двигатели
  42. Электрическое поле
  43. Постоянный ток
  44. Переменный ток
  45. Магнитное поле
  46. Электромагнитное поле
  47. Электромагнитное излучение
  48. Электрический заряд (Закон Кулона)
  49. Электрический ток в металлах
  50. Электрический ток в электролитах
  51. Электрический ток в газах и в вакууме
  52. Электрический ток в полупроводниках
  53. Электромагнитная индукция
  54. Работа, мощность и тепловое действие электрического тока
  55. Термоэлектрические явления
  56. Распространение электромагнитных волн
  57. Интерференционные явления
  58. Рассеяние
  59. Дифракция рентгеновских лучей на кристалле
  60. Двойное лучепреломление
  61. Магнитное поле и электромагнитная индукция
  62. Электромагнитные колебания и волны
  63. Природа света
  64. Распространение света
  65. Отражение и преломление света
  66. Оптические приборы и зрение
  67. Волновые свойства света
  68. Действия света
  69. Линзы и получение изображений с помощью линз
  70. Оптические приборы и глаз
  71. Фотометрия
  72. Излучение и спектры
  73. Квантовые свойства излучения
  74. Специальная теория относительности в физике
  75. Теория относительности
  76. Квантовая теория и природа поля
  77. Строение и свойства вещества
  78. Физика атомного ядра
  79. Строение атома

Давление. Единицы давления.

Человек на лыжах, и без них.

По рыхлому снегу человек идёт с большим трудом, глубоко проваливаясь при каждом шаге. Но, надев лыжи, он может идти, почти не проваливаясь в него. Почему? На лыжах или без лыж человек действует на снег с одной и той же силой, равной своему весу. Однако действие этой силы в обоих случаях различно, потому что различна площадь поверхности, на которую давит человек, с лыжами и без лыж. Площадь поверхности лыж почти в 20 раз больше площади подошвы. Поэтому, стоя на лыжах, человек действует на каждый квадратный сантиметр площади поверхности снега с силой, в 20 раз меньшей, чем стоя на снегу без лыж.

Ученик, прикалывая кнопками газету к доске, действует на каждую кнопку с одинаковой силой. Однако кнопка, имеющая более острый конец, легче входит в дерево.

Значит, результат действия силы зависит не только от её модуля, направления и точки приложения, но и от площади той поверхности, к которой она приложена (перпендикулярно которой она действует).

Этот вывод подтверждают физические опыты.

Опыт.Результат действия данной силы зависит от того, какая сила действует на единицу площади поверхности.

По углам небольшой доски надо вбить гвозди. Сначала гвозди, вбитые в доску, установим на песке остриями вверх и положим на доску гирю. В этом случае шляпки гвоздей лишь незначительно вдавливаются в песок. Затем доску перевернем и поставим гвозди на острие. В этом случае площадь опоры меньше, и под действием той же силы гвозди значительно углубляются в песок.

Опыт. Вторая иллюстрация.

От того, какая сила действует на каждую единицу площади поверхности, зависит результат действия этой силы.

В рассмотренных примерах силы действовали перпендикулярно поверхности тела. Вес человека был перпендикулярен поверхности снега; сила, действовавшая на кнопку, перпендикулярна поверхности доски.

Величина, равная отношению силы, действующей перпендикулярно поверхности, к площади этой поверхности, называется давлением.

Чтобы определить давление, надо силу, действующую перпендикулярно поверхности, разделить на площадь поверхности:

давление = сила / площадь.

Обозначим величины, входящие в это выражение: давление — p, сила, действующая на поверхность, — F и площадь поверхности — S.

Тогда получим формулу:

p = F/S

Понятно, что бóльшая по значению сила, действующую на ту же площадь, будет производить большее давление.

За единицу давления принимается такое давление, которое производит сила в 1 Н, действующая на поверхность площадью 1 м2 перпендикулярно этой поверхности.

Единица давления — ньютон на квадратный метр ( 1 Н / м2 ). В честь французского ученого Блеза Паскаля она называется паскалем (Па). Таким образом,

1 Па = 1 Н / м2 .

Используется также другие единицы давления: гектопаскаль (гПа) и килопаскаль (кПа).

1 кПа = 1000 Па;

1 гПа = 100 Па;

1 Па = 0,001 кПа;

1 Па = 0,01 гПа.

Пример. Рассчитать давление, производимое на пол мальчиком, масса которого 45 кг, а площадь подошв его ботинок, соприкасающихся с полом, равна 300 см2.

Запишем условие задачи и решим её.

Дано: m = 45 кг, S = 300 см2; p = ?

В единицах СИ: S = 0,03 м2

Решение:

p = F/S,

F = P,

P = g·m,

P = 9,8 Н · 45 кг ≈ 450 Н,

p = 450/0,03 Н / м2 = 15000 Па = 15 кПа

‘Ответ’: p = 15000 Па = 15 кПа

Способы уменьшения и увеличения давления.

Тяжелый гусеничный трактор производит на почву давление равное 40 — 50 кПа, т. е. всего в 2 — 3 раза больше, чем давление мальчика массой 45 кг. Это объясняется тем, что вес трактора распределяется на бóльшую площадь за счёт гусеничной передачи. А мы установили, что чем больше площадь опоры, тем меньше давление, производимое одной и той же силой на эту опору.

В зависимости от того, нужно ли получить малое или большое давление, площадь опоры увеличивается или уменьшается. Например, для того, чтобы грунт мог выдержать давление возводимого здания, увеличивают площадь нижней части фундамента.

Шины грузовых автомобилей и шасси самолетов делают значительно шире, чем легковых. Особенно широкими делают шины у автомобилей, предназначенных для передвижения в пустынях.

Тяжелые машины, как трактор, танк или болотоход, имея большую опорную площадь гусениц, проходят по болотистой местности, по которой не пройдет человек.

С другой стороны, при малой площади поверхности можно небольшой силой произвести большое давление. Например, вдавливая кнопку в доску, мы действуем на нее с силой около 50 Н. Так как площадь острия кнопки примерно 1 мм2, то давление, производимое ею, равно:

p = 50 Н/ 0, 000 001 м2 = 50 000 000 Па = 50 000 кПа.

Для сравнения, это давление в 1000 раз больше давления, производимого гусеничным трактором на почву. Можно найти еще много таких примеров.

Лезвие режущих и острие колющих инструментов (ножей, ножниц, резцов, пил, игл и др.) специально остро оттачивается. Заточенный край острого лезвия имеет маленькую площадь, поэтому при помощи даже малой силы создается большое давление, и таким инструментом легко работать.

Режущие и колющие приспособления встречаются и в живой природе: это зубы, когти, клювы, шипы и др. — все они из твердого материала, гладкие и очень острые.

Давление

Известно, что молекулы газа беспорядочно движутся.
Опыт. Здесь мы узнаем, что газ давит на стенки сосуда по всем направлениям одинаково.

Мы уже знаем, что газы, в отличие от твердых тел и жидкостей, заполняют весь сосуд, в котором находятся. Например, стальной баллон для хранения газов, камера автомобильной шины или волейбольный мяч. При этом газ оказывает давление на стенки, дно и крышку баллона, камеры или любого другого тела, в котором он находится. Давление газа обусловлено иными причинами, чем давление твердого тела на опору.

Известно, что молекулы газа беспорядочно движутся. При своем движении они сталкиваются друг с другом, а также со стенками сосуда, в котором находится газ. Молекул в газе много, поэтому и число их ударов очень велико. Например, число ударов молекул воздуха, находящегося в комнате, о поверхность площадью 1 см2 за 1 с выражается двадцатитрехзначным числом. Хотя сила удара отдельной молекулы мала, но действие всех молекул на стенки сосуда значительно, — оно и создает давление газа.

Итак, давление газа на стенки сосуда (и на помещенное в газ тело) вызывается ударами молекул газа.

Рассмотрим следующий опыт. Под колокол воздушного насоса поместим резиновый шарик. Он содержит небольшое количество воздуха и имеет неправильную форму. Затем насосом откачиваем воздух из-под колокола. Оболочка шарика, вокруг которой воздух становится все более разреженным, постепенно раздувается и принимает форму правильного шара.

Как объяснить этот опыт?

Для хранения и перевозки сжатого газа используются специальные прочные стальные баллоны.

В нашем опыте движущиеся молекулы газа непрерывно ударяют о стенки шарика внутри и снаружи. При откачивании воздуха число молекул в колоколе вокруг оболочки шарика уменьшается. Но внутри шарика их число не изменяется. Поэтому число ударов молекул о внешние стенки оболочки становится меньше, чем число ударов о внутренние стенки. Шарик раздувается до тех пор, пока сила упругости его резиновой оболочки не станет равной силе давления газа. Оболочка шарика принимает форму шара. Это показывает, что газ давит на ее стенки по всем направлениям одинаково. Иначе говоря, число ударов молекул, приходящихся на каждый квадратный сантиметр площади поверхности, по всем направлениям одинаково. Одинаковое давление по всем направлениям характерно для газа и является следствием беспорядочного движения огромного числа молекул.

Попытаемся уменьшить объем газа, но так, чтобы масса его осталась неизменной. Это значит, что в каждом кубическом сантиметре газа молекул станет больше, плотность газа увеличится. Тогда число ударов молекул о стенки увеличится, т. е. возрастет давление газа. Это можно подтвердить опытом.

На рисунке а изображена стеклянная трубка, один конец которой закрыт тонкой резиновой пленкой. В трубку вставлен поршень. При вдвигании поршня объем воздуха в трубке уменьшается, т. е. газ сжимается. Резиновая пленка при этом выгибается наружу, указывая на то, что давление воздуха в трубке увеличилось.

Наоборот, при увеличении объема этой же массы газа, число молекул в каждом кубическом сантиметре уменьшается. От этого уменьшится число ударов о стенки сосуда — давление газа станет меньше. Действительно, при вытягивании поршня из трубки объем воздуха увеличивается, пленка прогибается внутрь сосуда. Это указывает на уменьшение давления воздуха в трубке. Такие же явления наблюдались бы, если бы вместо воздуха в трубке находился бы любой другой газ.

Итак, при уменьшении объема газа его давление увеличивается, а при увеличении объема давление уменьшается при условии, что масса и температура газа остаются неизменными.

А как изменится давление газа, если нагреть его при постоянном объеме? Известно, что скорость движения молекул газа при нагревании увеличивается. Двигаясь быстрее, молекулы будут ударять о стенки сосуда чаще. Кроме того, каждый удар молекулы о стенку будет сильнее. Вследствие этого, стенки сосуда будут испытывать большее давление.

Следовательно, давление газа в закрытом сосуде тем больше, чем выше температура газа, при условии, что масса газа и объем не изменяются.

Из этих опытов можно сделать общий вывод, что давление газа тем больше, чем чаще и сильнее молекулы ударяют о стенки сосуда.

Для хранения и перевозки газов их сильно сжимают. При этом давление их возрастает, газы необходимо заключать в специальные, очень прочные баллоны. В таких баллонах, например, содержат сжатый воздух в подводных лодках, кислород, используемый при сварке металлов. Конечно же, мы должны навсегда запомнить, что газовые баллоны нельзя нагревать, тем более, когда они заполнены газом. Потому что, как мы уже понимаем, может произойти взрыв с очень неприятными последствиями.

Закон Паскаля.

Давление передается в каждую точку жидкости или газа.
Давление поршня передается в каждую точку жидкости, заполняющей шар.
Теперь газ.

В отличие от твердых тел отдельные слои и мелкие частицы жидкости и газа могут свободно перемещаться относительно друг друга по всем направлениям. Достаточно, например, слегка подуть на поверхность воды в стакане, чтобы вызвать движение воды. На реке или озере при малейшем ветерке появляется рябь.

Подвижностью частиц газа и жидкости объясняется, что давление, производимое на них, передается не только в направлении действия силы, а в каждую точку. Рассмотрим это явление подробнее.

На рисунке, а изображен сосуд, в котором содержится газ (или жидкость). Частицы равномерно распределены по всему сосуду. Сосуд закрыт поршнем, который может перемещаться вверх и вниз.

Прилагая некоторую силу, заставим поршень немного переместиться внутрь и сжать газ (жидкость), находящийся непосредственно под ним. Тогда частицы (молекулы) расположатся в этом месте более плотно, чем прежде(рис, б). Благодаря подвижности частицы газа будут перемещаться по всем направлениям. Вследствие этого их расположение опять станет равномерным, но более плотным, чем раньше (рис, в). Поэтому давление газа всюду возрастет. Значит, добавочное давление передается всем частицам газа или жидкости. Так, если давление на газ (жидкость) около самого поршня увеличится на 1 Па, то во всех точках внутри газа или жидкости давление станет больше прежнего на столько же. На 1 Па увеличится давление и на стенки сосуда, и на дно, и на поршень.

Давление, производимое на жидкость или газ, передается на любую точку одинаково во всех направлениях.

Это утверждение называется законом Паскаля.

На основе закона Паскаля легко объяснить следующие опыты.

На рисунке изображен полый шар, имеющий в различных местах небольшие отверстия. К шару присоединена трубка, в которую вставлен поршень. Если набрать воды в шар и вдвинуть в трубку поршень, то вода польется из всех отверстий шара. В этом опыте поршень давит на поверхность воды в трубке. Частицы воды, находящиеся под поршнем, уплотняясь, передают его давление другим слоям, лежащим глубже. Таким образом, давление поршня передается в каждую точку жидкости, заполняющей шар. В результате часть воды выталкивается из шара в виде одинаковых струек, вытекающих из всех отверстий.

Если шар заполнить дымом, то при вдвигании поршня в трубку из всех отверстий шара начнут выходить одинаковые струйки дыма. Это подтверждает, что и газы передают производимое на них давление во все стороны одинаково.

Давление в жидкости и газе.

Под действием веса жидкости резиновое дно в трубке прогнется.

На жидкости, как и на все тела на Земле, действует сила тяжести. Поэтому, каждый слой жидкости, налитой в сосуд, своим весом создает давление, которое по закону Паскаля передается по всем направлениям. Следовательно, внутри жидкости существует давление. В этом можно убедиться на опыте.

В стеклянную трубку, нижнее отверстие которой закрыто тонкой резиновой пленкой, нальем воду. Под действием веса жидкости дно трубки прогнется.

Опыт показывает, что, чем выше столб воды над резиновой пленкой, тем больше она прогибается. Но всякий раз после того, как резиновое дно прогнулось, вода в трубке приходит в равновесие (останавливается), так как, кроме силы тяжести, на воду действует сила упругости растянутой резиновой пленки.

Иллюстрация.
Дно отходит от цилиндра вследствие давления на него силы тяжести.

Опустим трубку с резиновым дном, в которую налита вода, в другой, более широкий сосуд с водой. Мы увидим, что по мере опускания трубки резиновая пленка постепенно выпрямляется. Полное выпрямление пленки показывает, что силы, действующие на нее сверху и снизу, равны. Наступает полное выпрямление пленки тогда, когда уровни воды в трубке и сосуде совпадают.

Такой же опыт можно провести с трубкой, в которой резиновая пленка закрывает боковое отверстие, как это показано на рисунке, а. Погрузим эту трубку с водой в другой сосуд с водой, как это изображено на рисунке, б. Мы заметим, что пленка снова выпрямится, как только уровни воды в трубке и сосуде сравняются. Это означает, что силы, действующие на резиновую пленку, одинаковы со всех сторон.

Возьмем сосуд, дно которого может отпадать. Опустим его в банку с водой. Дно при этом окажется плотно прижатым к краю сосуда и не отпадет. Его прижимает сила давления воды, направленная снизу вверх.

Будем осторожно наливать воду в сосуд и следить за его дном. Как только уровень воды в сосуде совпадет с уровнем воды в банке, оно отпадет от сосуда.

В момент отрыва на дно давит сверху вниз столб жидкости в сосуде, а снизу вверх на дно передается давление такого же по высоте столба жидкости, но находящейся в банке. Оба эти давления одинаковы, дно же отходит от цилиндра вследствие действия на него собственной силы тяжести.

Выше были описаны опыты с водой, но если взять вместо воды любую другую жидкость, результаты опыта будут те же.

Итак, опыты показывают, что внутри жидкости существует давление, и на одном и том же уровне оно одинаково по всем направлениям. С глубиной давление увеличивается.

Газы в этом отношении не отличаются от жидкостей, ведь они тоже имеют вес. Но надо помнить, что плотность газа в сотни раз меньше плотности жидкости. Вес газа, находящегося в сосуде, мал, и его «весовое» давление во многих случаях можно не учитывать.

Расчет давления жидкости на дно и стенки сосуда.

Расчет давления жидкости на дно и стенки сосуда.

Рассмотрим, как можно рассчитывать давление жидкости на дно и стенки сосуда. Решим сначала задачу для сосуда, имеющего форму прямоугольного параллелепипеда.

Сила F, с которой жидкость, налитая в этот сосуд, давит на его дно, равна весу P жидкости, находящейся в сосуде. Вес жидкости можно определить, зная ее массу m. Массу, как известно, можно вычислить по формуле: m = ρ·V. Объем жидкости, налитой в выбранный нами сосуд, легко рассчитать. Если высоту столба жидкости, находящейся в сосуде, обозначить буквой h, а площадь дна сосуда S, то V = S·h.

Масса жидкости m = ρ·V, или m = ρ·S·h .

Вес этой жидкости P = g·m, или P = g·ρ·S·h.

Так как вес столба жидкости равен силе, с которой жидкость давит на дно сосуда, то, разделив вес P на площадь S, получим давление жидкости p:

p = P/S , или p = g·ρ·S·h/S,

то есть

p = g·ρ·h.

Мы получили формулу для расчета давления жидкости на дно сосуда. Из этой формулы видно, что давление жидкости на дно сосуда зависит только от плотности и высоты столба жидкости.

Следовательно, по выведенной формуле можно рассчитывать давление жидкости, налитой в сосуд любой формы (строго говоря, наш расчет годится только для сосудов, имеющих форму прямой призмы и цилиндра. В курсах физики для института доказано, что формула верна и для сосуда произвольной формы). Кроме того, по ней можно вычислить и давление на стенки сосуда. Давление внутри жидкости, в том числе давление снизу вверх, также рассчитывается по этой формуле, так как давление на одной и той же глубине одинаково по всем направлениям.

При расчете давления по формуле p = gρh надо плотность ρ выражать в килограммах на кубический метр (кг/м3), а высоту столба жидкости h — в метрах (м), g = 9,8 Н/кг, тогда давление будет выражено в паскалях (Па).

Пример. Определите давление нефти на дно цистерны, если высота столба нефти 10 м, а плотность ее 800 кг/м3 .

Запишем условие задачи и запишем ее.

Дано:

h = 10 м

ρ = 800 кг/м3

P = ?

Решение:

p = gρh,

p = 9.8 Н/кг · 800 кг/м3 · 10 м ≈ 80 000 Па ≈ 80 кПа.

Ответ: p ≈ 80 кПа.

Сообщающиеся сосуды.

Сообщающиеся сосуды.

На рисунке изображены два сосуда, соединённые между собой резиновой трубкой. Такие сосуды называются сообщающимися. Лейка, чайник, кофейник — примеры сообщающихся сосудов. Из опыта мы знаем, что вода, налитая, например, в лейку, стоит всегда на одном уровне в носике и внутри.

С сообщающимися сосудами можно проделать следующий простой опыт. В начале опыта резиновую трубку зажимаем в середине, и в одну из трубок наливаем воду. Затем зажим открываем, и вода вмиг перетекает в другую трубку, пока поверхности воды в обеих трубках не установятся на одном уровне. Можно закрепить одну из трубок в штативе, а другую поднимать, опускать или наклонять в разные стороны. И в этом случае, как только жидкость успокоится, ее уровни в обеих трубках уравняются.

В сообщающихся сосудах любой формы и сечения поверхности однородной жидкости устанавливаются на одном уровне (при условии, что давление воздуха над жидкостью одинаково) (рис. 109).

Это можно обосновать следующим образом. Жидкость покоится, не перемещаясь из одного сосуда в другой. Значит, давления в обоих сосудах на любом уровне одинаковы. Жидкость в обоих сосудах одна и та же, т. е. имеет одинаковую плотность. Следовательно, должны быть одинаковы и ее высоты. Когда мы поднимаем один сосуд или доливаем в него жидкость, давление в нем увеличивается и жидкость перемещается в другой сосуд до тех пор, пока давления не уравновесятся.

Если в один из сообщающихся сосудов налить жидкость одной плотности, а во второй — другой плотности, то при равновесии уровни этих жидкостей не будут одинаковыми. И это понятно. Мы ведь знаем, что давление жидкости на дно сосуда прямо пропорционально высоте столба и плотности жидкости. А в этом случае плотности жидкостей будут различны.

При равенстве давлений высота столба жидкости с большей плотностью будет меньше высоты столба жидкости с меньшей плотностью (рис.).

Опыт. Как определить массу воздуха.

Вес воздуха. Атмосферное давление.

Существование атмосферного давления.
Атмосферное давление больше, чем давление разреженного воздуха в сосуде.

На воздух, как и на всякое тело, находящееся на Земле, действует сила тяжести, и, значит, воздух обладает весом. Вес воздуха легко вычислить, зная его массу.

На опыте покажем, как вычислить массу воздуха. Для этого нужно взять прочный стеклянный шар с пробкой и резиновой трубкой с зажимом. Выкачаем из него насосом воздух, зажмем трубку зажимом и уравновесим на весах. Затем, открыв зажим на резиновой трубке, впустим в него воздух. Равновесие весов при этом нарушится. Для его восстановления на другую чашку весов придется положить гири, масса которых будет равна массе воздуха в объеме шара.

Опытами установлено, что при температуре 0 °С и нормальном атмосферном давлении масса воздуха объемом 1 м3 равна 1,29 кг. Вес этого воздуха легко вычислить:

P = g·m, P = 9,8 Н/кг · 1,29 кг ≈ 13 Н.

Воздушная оболочка, окружающая Землю, называется атмосфера (от греч. атмос — пар, воздух, и сфера — шар).

Атмосфера, как показали наблюдения за полетом искусственных спутников Земли, простирается на высоту нескольких тысяч километров.

Вследствие действия силы тяжести верхние слои атмосферы, подобно воде океана, сжимают нижние слои. Воздушный слой, прилегающий непосредственно к Земле, сжат больше всего и, согласно закону Паскаля, передает производимое на него давление по всем направлениям.

В результате этого земная поверхность и телá, находящиеся на ней, испытывают давление всей толщи воздуха, или, как обычно говорится в таких случаях, испытывают атмосферное давление.

Существованием атмосферного давления могут быть объяснены многие явления, с которыми мы встречаемся в жизни. Рассмотрим некоторые из них.

На рисунке изображена стеклянная трубка, внутри которой находится поршень, плотно прилегающий к стенкам трубки. Конец трубки опущен воду. Если поднимать поршень, то за ним будет подниматься и вода.

Это явление используется в водяных насосах и некоторых других устройствах.

На рисунке показан цилиндрический сосуд. Он закрыт пробкой, в которую вставлена трубка с краном. Из сосуда насосом откачивается воздух. Затем конец трубки помещается в воду. Если теперь открыть кран, то вода фонтаном брызнет в внутрь сосуда. Вода поступает в сосуд потому, что атмосферное давление больше давления разреженного воздуха в сосуде.

Почему существует воздушная оболочка Земли.

Как и все тела, молекулы газов, входящих в состав воздушной оболочки Земли, притягиваются к Земле.

Но почему же тогда все они не упадут на поверхность Земли? Каким образом сохраняется воздушная оболочка Земли, ее атмосфера? Чтобы понять это, надо учесть, что молекулы газов находятся в непрерывном и беспорядочном движении. Но тогда возникает другой вопрос: почему эти молекулы не улетают в мировое пространство, то есть в космос.

Для того, чтобы совсем покинуть Землю, молекула, как и космический корабль или ракета, должна иметь очень большую скорость (не меньше 11,2 км/с). Это так называемая вторая космическая скорость. Скорость большинства молекул воздушной оболочки Земли значительно меньше этой космической скорости. Поэтому большинство их привязано к Земле силой тяжести, лишь ничтожно малое количество молекул улетает за пределы Земли в космос.

Беспорядочное движение молекул и действие на них силы тяжести приводят в результате к тому, что молекулы газов «парят» в пространстве около Земли, образуя воздушную оболочку, или известную нам атмосферу.

Измерения показывают, что плотность воздуха быстро уменьшается с высотой. Так, на высоте 5,5 км над Землей плотность воздуха в 2 раза меньше его плотность у поверхности Земли, на высоте 11 км — в 4 раза меньше, и т. д. Чем выше, тем воздух разреженнее. И наконец, в самых верхних слоях (сотни и тысячи километров над Землей) атмосфера постепенно переходит в безвоздушное пространство. Четкой границы воздушная оболочка Земли не имеет.

Строго говоря, вследствие действия силы тяжести плотность газа в любом закрытом сосуде неодинакова по всему объему сосуда. Внизу сосуда плотность газа больше, чем в верхних его частях, поэтому и давление в сосуде неодинаково. На дне сосуда оно больше, чем вверху.
Однако для газа, содержащегося в сосуде, это различие в плотности и давлении столь мало, что его можно во многих случаях совсем не учитывать, просто знать об этом. Но для атмосферы, простирающейся на несколько тысяч километров, различие это существенно.

Измерение атмосферного давления. Опыт Торричелли.

Рассчитать атмосферное давление по формуле для вычисления давления столба жидкости (§ 38) нельзя. Для такого расчета надо знать высоту атмосферы и плотность воздуха. Но определенной границы у атмосферы нет, а плотность воздуха на разной высоте различна. Однако измерить атмосферное давление можно с помощью опыта, предложенного в 17 веке итальянским ученым Эванджелиста Торричелли, учеником Галилея.

Опыт Торричелли состоит в следующем: стеклянную трубку длиной около 1 м, запаянную с одного конца, наполняют ртутью. Затем, плотно закрыв второй конец трубки, ее переворачивают и опускают в чашку с ртутью, где под уровнем ртути открывают этот конец трубки. Как и в любом опыте с жидкостью, часть ртути при этом выливается в чашку, а часть ее остается в трубке. Высота столба ртути, оставшейся в трубке, равна примерно 760 мм. Над ртутью внутри трубки воздуха нет, там безвоздушное пространство, поэтому никакой газ не оказывает давления сверху на столб ртути внутри этой трубки и не влияет на измерения.

Опыт Торричелли.

Торричелли, предложивший описанный выше опыт, дал и его объяснение. Атмосфера давит на поверхность ртути в чашке. Ртуть находится в равновесии. Значит, давление в трубке на уровне аа1 (см. рис) равно атмосферному давлению. При изменении атмосферного давления меняется и высота столба ртути в трубке. При увеличении давления столбик удлиняется. При уменьшении давления — столб ртути уменьшает свою высоту.

Давление в трубке на уровне аа1 создается весом столба ртути в трубке, так как в верхней части трубки над ртутью воздуха нет. Отсюда следует, что атмосферное давление равно давлению столба ртути в трубке, т. е.

pатм = pртути .

Измерив высоту столба ртути, можно рассчитать давление, которое производит ртуть. Оно и будет равно атмосферному давлению. Если атмосферное давление уменьшится, то столб ртути в трубке Торричелли понизится.

Чем больше атмосферное давление, тем выше столб ртути в опыте Торричелли. Поэтому на практике атмосферное давление можно измерить высотой ртутного столба (в миллиметрах или сантиметрах). Если, например, атмосферное давление равно 780 мм рт. ст. (говорят «миллиметров ртутного столба»), то это значит, что воздух производит такое же давление, какое производит вертикальный столб ртути высотой 780 мм.

Следовательно, в этом случае за единицу измерения атмосферного давления принимается 1 миллиметр ртутного столба (1 мм рт. ст.). Найдем соотношение между этой единицей и известной нам единицей — паскалем (Па).

Давление столба ртути ρртути высотой 1 мм равно:

p = g·ρ·h, p = 9,8 Н/кг · 13 600 кг/ м3 · 0,001 м ≈ 133,3 Па.

Итак, 1 мм рт. ст. = 133,3 Па.

В настоящее время атмосферное давление принято измерять в гектопаскалях ( 1 гПа = 100 Па). Например, в сводках погоды может быть объявлено, что давление равно 1013 гПа, это то же самое, что 760 мм рт. ст.

Наблюдая ежедневно за высотой ртутного столба в трубке, Торричелли обнаружил, что эта высота меняется, т. е. атмосферное давление непостоянно, оно может увеличиваться и уменьшаться. Торричелли заметил также, что атмосферное давление связано с изменением погоды.

Если к трубке с ртутью, использовавшейся в опыте Торричелли, прикрепить вертикальную шкалу, то получится простейший прибор — ртутный барометр (от греч. барос — тяжесть, метрео — измеряю). Он служит для измерения атмосферного давления.

Барометр — анероид.

В практике для измерения атмосферного давления используют металлический барометр, называемый анероидом (в переводе с греческого — безжидкостный). Так барометр называют потому, что в нем нет ртути.

Внешний вид анероида изображен на рисунке. Главная часть его — металлическая коробочка 1 с волнистой (гофрированной) поверхностью (см. др. рис.). Из этой коробочки выкачан воздух, а чтобы атмосферное давление не раздавило коробочку, ее крышка 2 пружиной оттягивается вверх. При увеличении атмосферного давления крышка прогибается вниз и натягивает пружину. При уменьшении давления пружина выпрямляет крышку. К пружине с помощью передаточного механизма 3 прикреплена стрелка-указатель 4, которая продвигается вправо или влево при изменении давления. Под стрелкой укреплена шкала, деления которой нанесены по показаниям ртутного барометра. Так, число 750, против которого стоит стрелка анероида (см. рис.), показывает, что в данный момент в ртутном барометре высота ртутного столба 750 мм.

Следовательно, атмосферное давление равно 750 мм рт. ст. или ≈ 1000 гПа.

Значение атмосферного давления весьма важно для предвидения погоды на ближайшие дни, так как изменение атмосферного давления связано с изменением погоды. Барометр — необходимый прибор для метеорологических наблюдений.

Атмосферное давление на различных высотах.

В жидкости давление, как мы знаем, зависит от плотности жидкости и высоты ее столба. Вследствие малой сжимаемости плотность жидкости на различных глубинах почти одинакова. Поэтому, вычисляя давление, мы считаем ее плотность постоянной и учитываем только изменение высоты.

Сложнее дело обстоит с газами. Газы сильно сжимаемы. А чем сильнее газ сжат, тем больше его плотность, и тем большее давление он производит. Ведь давление газа создается ударами его молекул о поверхность тела.

Слои воздуха у поверхности Земли сжаты всеми вышележащими слоями воздуха, находящимися над ними. Но чем выше от поверхности слой воздуха, тем слабее он сжат, тем меньше его плотность. Следовательно, тем меньшее давление он производит. Если, например, воздушный шар поднимается над поверхностью Земли, то давление воздуха на шар становиться меньше. Это происходит не только потому, что высота столба воздуха над ним уменьшается, но еще и потому, что уменьшается плотность воздуха. Вверху она меньше, чем внизу. Поэтому зависимость давления воздуха от высоты сложнее, чем жидкости.

Наблюдения показывают, что атмосферное давление в местностях, лежащих на уровне моря, в среднем равно 760 мм рт. ст.

Атмосферное давление, равное давлению столба ртути высотой 760 мм при температуре 0 °С, называется нормальным атмосферным давлением.

Нормальное атмосферное давление равно 101 300 Па = 1013 гПа.

Чем больше высота над уровнем моря, тем давление меньше.

При небольших подъемах, в среднем, на каждые 12 м подъема давление уменьшается на 1 мм рт. ст. (или на 1,33 гПа).

Зная зависимость давления от высоты, можно по изменению показаний барометра определить высоту над уровнем моря. Анероиды, имеющие шкалу, по которой непосредственно можно измерить высоту над уровнем моря, называются высотомерами. Их применяют в авиации и при подъеме на горы.

Манометры.

Мы уже знаем, что для измерения атмосферного давления применяют барометры. Для измерения давлений, бóльших или меньших атмосферного, используется манометры (от греч. манос — редкий, неплотный, метрео — измеряю). Манометры бывают жидкостные и металлические.

Рассмотрим сначала устройство и действие открытого жидкостного манометра. Он состоит из двухколенной стеклянной трубки, в которую наливается какая-нибудь жидкость. Жидкость устанавливается в обоих коленах на одном уровне, так как на ее поверхность в коленах сосуда действует только атмосферное давление.

Чтобы понять, как работает такой манометр, его можно соединить резиновой трубкой с круглой плоской коробкой, одна сторона которой затянута резиновой пленкой. Если надавить пальцем на пленку, то уровень жидкости в колене манометра, соединенном в коробкой, понизится, а в другом колене повысится. Чем это объясняется?

При надавливании на пленку увеличивается давление воздуха в коробке. По закону Паскаля это увеличение давления передается и жидкости в том колене манометра, которое присоединено к коробке. Поэтому давление на жидкость в этом колене будет больше, чем в другом, где на жидкость действует только атмосферное давление. Под действием силы этого избыточного давления жидкость начнет перемещаться. В колене со сжатым воздухом жидкость опустится, в другом — поднимется. Жидкость придет в равновесие (остановится), когда избыточное давление сжатого воздуха уравновесится давлением, которое производит избыточный столб жидкости в другом колене манометра.

Чем сильнее давить на пленку, тем выше избыточный столб жидкости, тем больше его давление. Следовательно, об изменении давления можно судить по высоте этого избыточного столба.

На рисунке показано, как таким манометром можно измерять давление внутри жидкости. Чем глубже погружается в жидкость трубочка, тем больше становится разность высот столбов жидкости в коленах манометра, тем, следовательно, и большее давление производит жидкость.

Если установить коробочку прибора на какой-нибудь глубине внутри жидкости и поворачивать ее пленкой вверх, вбок и вниз, то показания манометра при этом не будут меняется. Так и должно быть, ведь на одном и том же уровне внутри жидкости давление одинаково по всем направлениям.

На рисунке изображен металлический манометр. Основная часть такого манометра — согнутая в трубу металлическая трубка 1, один конец которой закрыт. Другой конец трубки с помощью крана 4 сообщается с сосудом, в котором измеряют давление. При увеличении давления трубка разгибается. Движение её закрытого конца при помощи рычага 5 и зубчатки 3 передается стрелке 2, движущейся около шкалы прибора. При уменьшении давления трубка, благодаря своей упругости, возвращается в прежнее положение, а стрелка — к нулевому делению шкалы.

Поршневой жидкостный насос.

В опыте, рассмотренном нами ранее (§ 40), было установлено, что вода в стеклянной трубке под действием атмосферного давления поднималась вверх за поршнем. На этом основано действие поршневых насосов.

Насос схематически изображен на рисунке. Он состоит из цилиндра, внутри которого ходит вверх и вниз, плотно прилегая к стенкам сосуда, поршень 1. В нижней части цилиндра и в самом поршне установлены клапаны 2, открывающиеся только вверх. При движении поршня вверх вода под действием атмосферного давления входит в трубу, поднимает нижний клапан и движется за поршнем.

При движении поршня вниз вода, находящаяся под поршнем, давит на нижний клапан, и он закрывается. Одновременно под давлением воды открывается клапан внутри поршня, и вода переходит в пространство над поршнем. При следующем движении поршня вверх в месте с ним поднимается и находящаяся над ним вода, которая и выливается в отводящую трубу. Одновременно за поршнем поднимается и новая порция воды, которая при последующем опускании поршня окажется над ним, и вся эта процедура повторяется вновь и вновь, пока работает насос.

Гидравлический пресс.

Закон Паскаля позволяет объяснить действие гидравлической машины (от греч. гидравликос — водяной). Это машины, действие которых основано на законах движения и равновесия жидкостей.

Основной частью гидравлической машины служат два цилиндра разного диаметра, снабженные поршнями и соединительной трубкой. Пространство под поршнями и трубку заполняют жидкостью (обычно минеральным маслом). Высоты столбов жидкости в обоих цилиндрах одинаковы, пока на поршни не действуют силы.

Допустим теперь, что силы F1 и F2 — силы, действующие на поршни, S1 и S2 — площади поршней. Давление под первым (малым) поршнем равно p1 = F1 / S1, а под вторым (большим) p2 = F2 / S2 . По закону Паскаля давление покоящейся жидкостью во все стороны передается одинаково, т. е. p1 = p2 или F1 / S1 = F2 / S2 , откуда:

F2 / F1 = S2 / S1 .

Следовательно, сила F2 во столько раз больше силы F1 , во сколько раз площадь большого поршня больше площади малого поршня. Например, если площадь большого поршня 500 см2, а малого 5 см2, и на малый поршень действует сила 100 Н, то на больший поршень будет действовать сила, в 100 раз бóльшая, то есть 10 000 Н.

Таким образом, с помощью гидравлической машины можно малой силой уравновесить бóльшую силу.

Отношение F1 / F2 показывает выигрыш в силе. Например, в приведенном примере выигрыш в силе равен 10 000 Н / 100 Н = 100.

Гидравлическая машина, служащая для прессования (сдавливания), называется гидравлическим прессом.

Гидравлические прессы применяются там, где требуется большая сила. Например, для выжимания масла из семян на маслобойных заводах, для прессования фанеры, картона, сена. На металлургических заводах гидравлические прессы используют для изготовления стальных валов машин, железнодорожных колес и многих других изделий. Современные гидравлические прессы могут развивать силу в десятки и сотни миллионов ньютонов.

Устройство гидравлического пресса схематически показано на рисунке. Прессуемое тело 1 (A) кладут на платформу, соединенную с большим поршнем 2 (B). При помощи малого поршня 3 (D) создается большое давление на жидкость. Это давление передается в каждую точку жидкости, заполняющей цилиндры. Поэтому такое же давление действует и на второй, большой поршень. Но так как площадь 2-го (большого) поршня больше площади малого, то и сила, действующая на него, будет больше силы, действующей на поршень 3 (D). Под действием этой силы поршень 2 (B) будет подниматься. При подъеме поршня 2 (B) тело (A) упирается в неподвижную верхнюю платформу и сжимается. При помощи манометра 4 (M) измеряется давление жидкости. Предохранительный клапан 5 (P) автоматически открывается, когда давление жидкости превышает допустимое значение.

Из малого цилиндра в большой жидкость перекачивается повторными движениями малого поршня 3 (D). Это осуществляется следующим образом. При подъеме малого поршня (D) клапан 6 (K) открывается, и в пространство, находящееся под поршнем, засасывается жидкость. При опускании малого поршня под действием давления жидкости клапан 6 (K) закрывается, а клапан 7 (K’) открывается, и жидкость переходит в большой сосуд.

Действие воды и газа на погруженное в них тело.

Под водой мы легко можем поднять камень, который с трудом поднимается в воздухе. Если погрузить пробку под воду и выпустить ее из рук, то она всплывет. Как можно объяснить эти явления?

Мы знаем (§ 38), что жидкость давит на дно и стенки сосуда. И если внутрь жидкости поместить какое-нибудь твердое тело, то оно также будет подвергаться давлению, как и стенки сосуда.

Рассмотрим силы, которые действуют со стороны жидкости на погруженное в нее тело. Чтобы легче было рассуждать, выберем тело, которое имеет форму параллелепипеда с основаниями, параллельными поверхности жидкости (рис.). Силы, действующие на боковые грани тела, попарно равны и уравновешивают друг друга. Под действием этих сил тело сжимается. А вот силы, действующие на верхнюю и нижнюю грани тела, неодинаковы. На верхнюю грань давит сверху силой F1 столб жидкости высотой h1 . На уровне нижней грани давление производит столб жидкости высотой h2. Это давление, как мы знаем (§ 37), передается внутри жидкости во все стороны. Следовательно, на нижнюю грань тела снизу вверх с силой F2 давит столб жидкости высотой h2. Но h2 больше h1, следовательно, и модуль силы F2 больше модуля силы F1. Поэтому тело выталкивается из жидкости с силой Fвыт, равной разности сил F2F1 , т. е.

Fвыт = F2F1

Рассчитаем эту выталкивающую силу. Силы F1 и F2 , действующие на верхнюю и нижнюю грани параллелепипеда, можно вычислить, зная площади этих граней (S1 и S2) и давление жидкости на уровнях этих граней (p1 и p2):

F1 = p1·S1, а F2 = p2·S2, так как p1 = ρж·g·h1 , p2 = ρж·g·h2 , а S1 = S2 = S, где S — площадь грани параллелепипеда (все грани равны).

Тогда, Fвыт = F2 — F1 = ρ·g·h2·S — ρ·g·h1·S = ρ·g·S·(h2 — h1) = ρ·g·S·h, где h — высота параллелепипеда (h = h2 — h1).

Но S·h = V, где V — объем параллелепипеда, а ρж·V = mж — масса жидкости в объеме параллелепипеда. Следовательно,

Fвыт = g·mж = Pж ,

т. е. выталкивающая сила равна весу жидкости в объеме погруженного в нее тела (выталкивающая сила равна весу жидкости такого же объёма, как и объём погруженного в нее тела).

Существование силы, выталкивающей тело из жидкости, легко обнаружить на опыте.

На рисунке а изображено тело, подвешенное к пружине со стрелкой-указателем на конце. Стрелка отмечает на штативе растяжение пружины. При отпускании тела в воду пружина сокращается (рис., б). Такое же сокращение пружины получится, если действовать на тело снизу вверх с некоторой силой, например, нажать рукой (приподнять).

Следовательно, опыт подтверждает, что на тело, находящееся в жидкости, действует сила, выталкивающая это тело из жидкости.

К газам, как мы знаем, также применим закон Паскаля. Поэтому на тела, находящиеся в газе, действует сила, выталкивающая их из газа. Под действием этой силы воздушные шары поднимаются вверх. Существование силы, выталкивающей тело из газа, можно также наблюдать на опыте.

К укороченной чашке весов подвесим стеклянный шар или большую колбу, закрытую пробкой. Весы уравновешиваются. Затем под колбу (или шар) ставят широкий сосуд так, чтобы он окружал всю колбу. Сосуд наполняется углекислым газом, плотность которого больше плотности воздуха (поэтому углекислый газ опускается вниз и заполняет сосуд, вытесняя из него воздух). При этом равновесие весов нарушается. Чашка с подвешенной колбой поднимается вверх (рис.). На колбу, погруженную в углекислый газ, действует бóльшая выталкивающая сила, по сравнению с той, которая действует на нее в воздухе.

Сила, выталкивающая тело из жидкости или газа, направлена противоположно силе тяжести, приложенной к этому телу.

Поэтому пролкосмосе). Именно этим объясняется, что в воде мы иногда легко поднимаем тела, которые с трудом удерживаем в воздухе.

Архимедова сила.

Силу, с которой тело, находящееся в жидкости, выталкивается ею, можно рассчитать (как это сделано в § 48). А можно определить ее значение на опыте, используя для этого прибор, изображенный на рисунке.

К пружине подвешивается небольшое ведерко и тело цилиндрической формы (рис., а). Стрелка на штативе отмечает растяжение пружины. Она показывает вес тела в воздухе. Приподняв тело, под него подставляется отливной сосуд, наполненный жидкостью до уровня отливной трубки. После чего тело погружается целиком в жидкость (рис., б). При этом часть жидкости, объем которой равен объему тела, выливается из отливного сосуда в стакан. Пружина сокращается, и указатель пружины поднимается вверх, показывая уменьшение веса тела в жидкости. В данном случае на тело, кроме силы тяжести, действует еще одна сила, выталкивающая его из жидкости. Если в верхнее ведерко вылить жидкость из стакана (т. е. ту, которую вытеснило тело), то указатель пружины возвратится к своему начальному положению (рис., в).

На основании этого опыта можно заключить, что сила, выталкивающая целиком погруженное в жидкость тело, равна весу жидкости в объеме этого тела. Такой же вывод мы получили и в § 48.

Если подобный опыт проделать с телом, погруженным в какой-либо газ, то он показал бы, что сила, выталкивающая тело из газа, также равна весу газа, взятого в объеме тела.

Сила, выталкивающая тело из жидкости или газа, называется архимедовой силой, в честь ученого Архимеда, который впервые указал на ее существование и рассчитал ее значение.

Итак, опыт подтвердил, что архимедова (или выталкивающая) сила равна весу жидкости в объеме тела, т. е. FА = Pж = g·mж. Массу жидкости mж, вытесняемую телом, можно выразить через ее плотность ρж и объем тела Vт, погруженного в жидкость (так как Vж — объем вытесненной телом жидкости равен Vт — объему тела, погруженного в жидкость), т. е. mж = ρж·Vт. Тогда получим:

FA = g·ρж·Vт

Следовательно, архимедова сила зависит от плотности жидкости, в которую погружено тело, и от объема этого тела. Но она не зависит, например, от плотности вещества тела, погружаемого в жидкость, так как эта величина не входит в полученную формулу.

Определим теперь вес тела, погруженного в жидкость (или в газ). Так как две силы, действующие на тело в этом случае, направлены в противоположные стороны (сила тяжести вниз, а архимедова сила вверх), то вес тела в жидкости P1 будет меньше веса тела в вакууме P = g·m на архимедову силу FА = g·mж (где mж — масса жидкости или газа, вытесненной телом).

Таким образом, если тело погружено в жидкость или газ, то оно теряет в своем весе столько, сколько весит вытесненная им жидкость или газ.

Пример. Определить выталкивающую силу, действующую на камень объемом 1,6 м3 в морской воде.

Запишем условие задачи и решим ее.

Дано:

Vт =1,6 м3

ρж = 1030 кг/м3

g = 9,8 Н/кг

FА — ?

Решение:

FА = g·ρж·Vт,

FА = 9.8 Н/кг · 1030 кг/м3 · 1,6 м3 = 16 480 Н ≈ 16,5 кН.

Ответ: FА = 16,5 кН.

Плавание тел.

На тело, находящееся внутри жидкости, действуют две силы: сила тяжести, направленная вертикально вниз, и архимедова сила, направленная вертикально вверх. Рассмотрим, что будет происходить с телом под действием этих сил, если в начале оно было неподвижно. При этом возможны три случая:

1) если сила тяжести Fтяж больше архимедовой силы FА, то тело будет опускаться на дно, тонуть, т. е. если

Fтяж > FА, то тело тонет;

2) если сила тяжести равна архимедовой силе, то тело может находиться в равновесии в любом месте жидкости, т. е. если

Fтяж = FА, то тело плавает;

3) если сила тяжести меньше архимедовой силы, то тело будет подниматься из жидкости, всплывать, т. е. если

Fтяж < FА, то тело всплывает.

Рассмотрим последний случай подробнее.

Когда всплывающее тело достигнет поверхности жидкости, то при дальнейшем его движении вверх архимедова сила будет уменьшаться. Почему? А потому, что будет уменьшаться объем части тела, погруженной в жидкость, а архимедова сила равна весу жидкости в объеме погруженной в нее части тела.

Когда архимедова сила станет равной силе тяжести, тело остановится и будет плавать на поверхности жидкости, частично погрузившись в нее.

Полученный вывод легко проверить на опыте.

В отливной сосуд нальем воду до уровня отливной трубки. После этого погрузим в сосуд плавающее тело, предварительно взвесив его в воздухе. Опустившись в воду, тело вытесняет объем воды, равный объему погруженной в нее части тела. Взвесив эту воду, находим, что ее вес (архимедова сила) равен силе тяжести, действующей на плавающее тело, или весу этого тела в воздухе.

Проделав такие же опыты с любыми другими телами, плавающими в разных жидкостях — в воде, спирте, растворе соли, можно убедиться, что если тело плавает в жидкости, то вес вытесненной им жидкости равен весу этого тела в воздухе.

Легко доказать, что если плотность сплошного твердого тела больше плотности жидкости, то тело в такой жидкости тонет. Тело с меньшей плотностью всплывает в этой жидкости. Кусок железа, например, тонет в воде, но всплывает в ртути. Тело же, плотность которого равна плотности жидкости, остается в равновесии внутри жидкости.

Плавает на поверхности воды лед, так как его плотность меньше плотности воды.

Чем меньше плотность тела по сравнению с плотностью жидкости, тем меньшая часть тела погружена в жидкость.

При равных плотностях тела и жидкости тело плавает внутри жидкости на любой глубине.

Две несмешивающиеся жидкости, например вода и керосин, располагаются в сосуде в соответствии со своими плотностями: в нижней части сосуда — более плотная вода (ρ = 1000 кг/м3), сверху — более легкий керосин (ρ = 800 кг/м3).

Средняя плотность живых организмов, населяющих водную среду, мало отличается от плотности воды, поэтому их вес почти полностью уравновешивается архимедовой силой. Благодаря этому водные животные не нуждаются в столь прочных и массивных скелетах, как наземные. По этой же причине эластичны стволы водных растений.

Плавательный пузырь рыбы легко меняет свой объем. Когда рыба с помощью мышц опускается на большую глубину, и давление воды на нее увеличивается, пузырь сжимается, объем тела рыбы уменьшается, и она не выталкивается вверх, а плавает в глубине. Таким образом, рыба может в определенных пределах регулировать глубину своего погружения. Киты регулируют глубину своего погружения за счет уменьшения и увеличения объема легких.

Плавание судов.

Суда, плавающие по рекам, озерам, морям и океанам, построены из разных материалов с различной плотностью. Корпус судов обычно делается из стальных листов. Все внутренние крепления, придающие судам прочность, также изготовляют из металлов. Для постройки судов используют различные материалы, имеющие по сравнению с водой как бóльшие, так и меньшие плотности.

Благодаря чему суда держатся на воде, принимают на борт и перевозят большие грузы?

Опыт с плавающим телом (§ 50) показал, что тело вытесняет своей подводной частью столько воды, что по весу эта вода равна весу тела в воздухе. Это также справедливо и для любого судна.

Вес воды, вытесняемой подводной частью судна, равен весу судна с грузом в воздухе или силе тяжести, действующей на судно с грузом.

Глубина, на которую судно погружается в воду, называется осадкой. Наибольшая допускаемая осадка отмечена на корпусе судна красной линией, называемой ватерлинией (от голланд. ватер — вода).

Вес воды, вытесняемой судном при погружении до ватерлинии, равный силе тяжести, действующей на судно с грузом, называется водоизмещением судна.

В настоящее время для перевозки нефти строятся суда водоизмещением 5 000 000 кН (5 · 106 кН) и больше, т. е. имеющие вместе с грузом массу 500 000 т (5 · 105 т) и более.

Если из водоизмещения вычесть вес самого судна, то мы получим грузоподъемность этого судна. Грузоподъемность показывает вес груза, перевозимого судном.

Судостроение существовало еще в Древнем Египте, в Финикии (считается, что Финикийцы были одними из лучших судостроителей), Древнем Китае.

В России судостроение зародилось на рубеже 17-18 вв. Сооружались главным образом военные корабли, но именно в России были построены первый ледокол, суда с двигателем внутреннего сгорания, атомный ледокол «Арктика».

Воздухоплавание.

Рисунок с описанием шара братьев Монгольфье 1783 года: «Вид и точные размеры „Аэростата Земной шар“, который был первым». 1786

С давних времен люди мечтали о возможности летать над облаками, плавать в воздушном океане, как они плавали по морю. Для воздухоплавания

вначале использовали воздушные шары, которые наполняли или нагретым воздухом, или водородом либо гелием.

Для того, чтобы воздушный шар поднялся в воздух, необходимо, чтобы архимедова сила (выталкивающая) FА, действующая на шар, была больше силы тяжести Fтяж, т. е. FА > Fтяж.

По мере поднятия шара вверх архимедова сила, действующая на него, уменьшается (FА = gρV), так как плотность верхних слоев атмосферы меньше, чем у поверхности Земли. Чтобы подняться выше, с шара сбрасывается специальный балласт (груз) и этим облегчает шар. В конце концов шар достигает своей своей предельной высоты подъема. Для спуска шара из его оболочки при помощи специального клапана выпускается часть газа.

В горизонтальном направлении воздушный шар перемещается только под действием ветра, поэтому он называется аэростатом (от греч аэр — воздух, стато — стоящий). Для исследования верхних слоев атмосферы, стратосферы еще не так давно применялись огромные воздушные шары — стратостаты.

До того как научились строить большие самолеты для перевозки по воздуху пассажиров и грузов, применялись управляемые аэростаты — дирижабли. Они имеют удлиненную форму, под корпусом подвешивается гондола с двигателем, который приводит в движение пропеллер.

Воздушный шар не только сам поднимается вверх, но может поднять и некоторый груз: кабину, людей, приборы. Поэтому для того, чтобы узнать, какой груз может поднять воздушный шар, необходимо определить его подъемную силу.

Пусть, например, в воздух запущен шар объемом 40 м3, наполненный гелием. Масса гелия, заполняющая оболочку шара, будет равна:
mГе = ρГе·V = 0,1890 кг/м3 · 40 м3 = 7,2 кг,
а его вес равен:
PГе = g·mГе ; PГе = 9,8 Н/кг · 7,2 кг = 71 Н.
Выталкивающая же сила (архимедова), действующая на этот шар в воздухе, равна весу воздуха объемом 40 м3, т. е.
FА = g·ρвоздV; FА = 9,8 Н/кг · 1,3 кг/м3 · 40 м3 = 520 Н.

Значит, этот шар может поднять груз весом 520 Н — 71 Н = 449 Н. Это и есть его подъемная сила.

Шар такого же объема, но наполненный водородом, может поднять груз 479 Н. Значит, подъемная сила его больше, чем шара, наполненного гелием. Но все же чаще используют гелий, так как он не горит и поэтому безопаснее. Водород же горючий газ.

Гораздо проще осуществить подъем и спуск шара, наполненного горячим воздухом. Для этого под отверстием, находящимся в нижней части шара, располагается горелка. При помощи газовой горелки можно регулировать температуру воздуха внутри шара, а значит, его плотность и выталкивающую силу. Чтобы шар поднялся выше, достаточно сильнее нагреть воздух в нем, увеличив пламя горелки. При уменьшении пламени горелки температура воздуха в шаре уменьшается, и шар опускается вниз.

Можно подобрать такую температуру шара, при которой вес шара и кабины будет равен выталкивающей силе. Тогда шар повиснет в воздухе, и с него будет легко проводить наблюдения.

По мере развития науки происходили и существенные изменения в воздухоплавательной технике. Появилась возможность использования новых оболочек для аэростатов, которые стали прочными, морозоустойчивыми и легкими.

Достижения в области радиотехники, электроники, автоматики позволили сконструировать беспилотные аэростаты. Эти аэростаты используются для изучения воздушных течений, для географических и медико-биологических исследований в нижних слоях атмосферы.

Ссылки

  • Уроки по физике за 7 класс по школьной программе

Давление газа обусловлено иными причинами, чем давление твёрдого тела на опору.

Расстояния между молeкулами газа значительно больше. Двигаясь хаотично, молекулы сталкиваются между собой и ударяют о стенки занимаемого им сосуда. Давление газа на стенки сосуда и вызывается этими ударами молекул газа.

сосуд.svg

Рис. (1). Газ в сосуде

Обрати внимание!

Давление газа тем больше, чем чаще и сильнее молекулы ударяют о стенки сосуда.

Для газа характерно одинаковое давление по всем направлениям, оно является следствием беспорядочного движения огромного числа молекул.

Давление газа на внутренние поверхности (дно, крышку, стенки) сосуда, в который он помещён, одинаково по всем направлениям.

шарик.svg

Рис. (2). Газ в воздушном шаре

Все воздушные шары приобретают форму, в которой давление равномерно растягивает стенки шара. Сфера (шар) — форма, в которой давление на поверхность имеет наименьшее значение и равномерно по всем направлениям.

Сжатые газы проще транспортировать. Плотность сжатых газов больше, давление намного больше. Поэтому используют прочные сосуды — стальные баллоны.

Сжатый воздух используется для дайвинга. Горючие газы удобнее хранить в сжатом виде.

Рис. (3). Сжатые газы

Свойства

1. При уменьшении объёма газа его давление увеличивается, а при увеличении объёма — давление уменьшается (при условии, что масса и температура газа остаются неизменными).

2. Давление газа в закрытом сосуде тем больше, чем выше температура газа (при условии, что масса газа и объём не изменяются).

сосуд подогрев.png

Рис. (4). Подогрев газа в сосуде

3. При увеличении массы газа давление увеличивается и наоборот.

Источники:

Рис. 1. Газ в сосуде.

Рис. 2. Газ в воздушном шаре.

Рис. 4. Подогрев газа в сосуде.

Горелка. Указание авторства не требуется, 2021-07-29. Pixabay License, https://pixabay.com/images/id-3053616/.

Изопроцессы

  • Темы кодификатора ЕГЭ: изопроцессы — изотермический, изохорный, изобарный процессы.

  • Изотермический процесс

  • Графики изотермического процесса

  • Изобарный процесс

  • Графики изобарного процесса

  • Изохорный процесс

  • Графики изохорного процесса

Автор статьи — профессиональный репетитор, автор учебных пособий для подготовки к ЕГЭ Игорь Вячеславович Яковлев

Темы кодификатора ЕГЭ: изопроцессы — изотермический, изохорный, изобарный процессы.

На протяжении этого листка мы будем придерживаться следующего предположения: масса и химический состав газа остаются неизменными. Иными словами, мы считаем, что:

m = const, то есть нет утечки газа из сосуда или, наоборот, притока газа в сосуд;

mu = const, то есть частицы газа не испытывают каких-либо изменений (скажем, отсутствует диссоциация — распад молекул на атомы).

Эти два условия выполняются в очень многих физически интересных ситуациях (например, в простых моделях тепловых двигателей) и потому вполне заслуживают отдельного рассмотрения.

Если масса газа и его молярная масса фиксированы, то состояние газа определяется тремя макроскопическими параметрами: давлением, объёмом и температурой. Эти параметры связаны друг с другом уравнением состояния (уравнением Менделеева — Клапейрона).

Термодинамический процесс (или просто процесс) — это изменение состояния газа с течением времени. В ходе термодинамического процесса меняются значения макроскопических параметров — давления, объёма и температуры.

Особый интерес представляют изопроцессы — термодинамические процессы, в которых значение одного из макроскопических параметров остаётся неизменным. Поочерёдно фиксируя каждый из трёх параметров, мы получим три вида изопроцессов.

1. Изотермический процесс идёт при постоянной температуре газа: T = const.
2. Изобарный процесс идёт при постоянном давлении газа: p = const.
3. Изохорный процесс идёт при постоянном объёме газа: V = const.

Изопроцессы описываются очень простыми законами Бойля — Мариотта, Гей-Люссака и Шарля. Давайте перейдём к их изучению.

к оглавлению ▴

Изотермический процесс

Пусть идеальный газ совершает изотермический процесс при температуре T. В ходе процесса меняются только давление газа и его объём.

Рассмотрим два произвольных состояния газа: в одном из них значения макроскопических параметров равны p_1, V_1, T, а во втором — p_2, V_2, T. Эти значения связаны уравнением Менделеева-Клапейрона:

p_1V_1=frac{displaystyle m}{displaystyle mu vphantom{1^a}}RT,

p_2V_2=frac{displaystyle m}{displaystyle mu vphantom{1^a}}RT.

Как мы сказали с самого начала,масса m и молярная масса mu предполагаются неизменными.

Поэтому правые части выписанных уравнений равны. Следовательно, равны и левые части:

p_1V_1=p_2V_2. (1)

Поскольку два состояния газа были выбраны произвольно, мы можем заключить, что в ходе изотермического процесса произведение давления газа на его объём остаётся постоянным:

pV = const. (2)

Данное утверждение называется законом Бойля — Мариотта.

Записав закон Бойля — Мариотта в виде

p=frac{displaystyle const}{displaystyle V vphantom{1^a}}, (3)

можно дать и такую формулировку: в изотермическом процессе давление газа обратно пропорционально его объёму. Если, например, при изотермическом расширении газа его объём увеличивается в три раза, то давление газа при этом в три раза уменьшается.

Как объяснить обратную зависимость давления от объёма с физической точки зрения? При постоянной температуре остаётся неизменной средняя кинетическая энергия молекул газа, то есть, попросту говоря, не меняется сила ударов молекул о стенки сосуда. При увеличении объёма концентрация молекул уменьшается, и соответственно уменьшается число ударов молекул в единицу времени на единицу площади стенки — давление газа падает. Наоборот, при уменьшении объёма концентрация молекул возрастает, их удары сыпятся чаще и давление газа увеличивается.

к оглавлению ▴

Графики изотермического процесса

Вообще, графики термодинамических процессов принято изображать в следующих системах координат:

pV-диаграмма: ось абсцисс V, ось ординат p;
VT-диаграмма: ось абсцисс T, ось ординат V;
pT-диаграмма: ось абсцисс T, ось ординат p.

График изотермического процесса называется изотермой.

Изотерма на pV-диаграмме — это график обратно пропорциональной зависимости p=frac{displaystyle const}{displaystyle V vphantom{1^a}}.

Такой график является гиперболой (вспомните алгебру — график функции y=frac{displaystyle k}{displaystyle x vphantom{1^a}}). Изотерма-гипербола изображена на рис. 1.

Рис. 1. Изотерма на pV-диаграмме

Каждая изотерма отвечает определённому фиксированному значению температуры. Оказывается, что чем выше температура, тем выше лежит соответствующая изотерма на pVдиаграмме.

В самом деле, рассмотрим два изотермических процесса, совершаемых одним и тем же газом (рис. 2). Первый процесс идёт при температуре T_1, второй — при температуре T_2.

Рис. 2. Чем выше температура, тем выше изотерма

Фиксируем некоторое значение объёма V. На первой изотерме ему отвечает давление p_1, на второй — p_2 > p_1. Но при фиксированном объёме давление тем больше, чем выше температура (молекулы начинают сильнее бить по стенкам). Значит, T_2 > T_1.

В оставшихся двух системах координат изотерма выглядит очень просто: это прямая, перпендикулярная оси T (рис. 3):

Рис. 3. Изотермы на VT и pT-диаграммах

к оглавлению ▴

Изобарный процесс

Напомним ещё раз, что изобарный процесс — это процесс, проходящий при постоянном давлении. В ходе изобарного процесса меняются лишь объём газа и его температура.

Типичный пример изобарного процесса: газ находится под массивным поршнем, который может свободно перемещаться. Если масса поршня M и поперечное сечение поршня S, то давление газа всё время постоянно и равно

p=p_0 + frac{displaystyle Mg}{displaystyle S vphantom{1^a}},

где p_0 — атмосферное давление.

Пусть идеальный газ совершает изобарный процесс при давлении p. Снова рассмотрим два произвольных состояния газа; на этот раз значения макроскопических параметров будут равны p, V_1, T_1 и p, V_2, T_2.

Выпишем уравнения состояния:

pV_1=frac{displaystyle m}{displaystyle mu vphantom{1^a}}RT_1,

pV_2=frac{displaystyle m}{displaystyle mu vphantom{1^a}}RT_2.

Поделив их друг на друга, получим:

frac{displaystyle V_1}{displaystyle V_2 vphantom{1^a}}=frac{displaystyle T_1}{displaystyle T_2 vphantom{1^a}}.

В принципе, уже и этого могло бы быть достаточно, но мы пойдём немного дальше. Перепишем полученное соотношение так, чтобы в одной части фигурировали только параметры первого состояния, а в другой части — только параметры второго состояния (иными словами, «разнесём индексы» по разным частям):

frac{displaystyle V_1}{displaystyle T_1 vphantom{1^a}}=frac{displaystyle V_2}{displaystyle T_2 vphantom{1^a}}. (4)

А отсюда теперь — ввиду произвольности выбора состояний! — получаем закон Гей-Люссака:

frac{displaystyle V}{displaystyle T vphantom{1^a}}=const. (5)

Иными словами, при постоянном давлении газа его объём прямо пропорционален температуре:

V=const cdot T. (6)

Почему объём растёт с ростом температуры? При повышении температуры молекулы начинают бить сильнее и приподнимают поршень. При этом концентрация молекул падает, удары становятся реже, так что в итоге давление сохраняет прежнее значение.

к оглавлению ▴

Графики изобарного процесса

График изобарного процесса называется изобарой. На VT-диаграмме изобара V = const cdot T является прямой линией (рис. 4):

Рис. 4. Изобара на VT-диаграмме

Пунктирный участок графика означает, что в случае реального газа при достаточно низких температурах модель идеального газа (а вместе с ней и закон Гей-Люссака) перестаёт работать. В самом деле, при снижении температуры частицы газа двигаются всё медленнее, и силы межмолекулярного взаимодействия оказывают всё более существенное влияние на их движение (аналогия: медленный мяч легче поймать, чем быстрый). Ну а при совсем уж низких температурах газы и вовсе превращаются в жидкости.

Разберёмся теперь, как меняется положение изобары при изменении давления. Оказывается, что чем больше давление, тем ниже идёт изобара на VTдиаграмме.
Чтобы убедиться в этом, рассмотрим две изобары с давлениями p_1 и p_2 (рис. 5):

Рис. 5. Чем ниже изобара, тем больше давление

Зафиксируем некоторое значение температуры T. Мы видим, что V_2 < V_1. Но при фиксированной температуре объём тем меньше, чем больше давление (закон Бойля — Мариотта!).

Стало быть, p_2 > p_1.

В оставшихся двух системах координат изобара является прямой линией, перпендикулярной оси p(рис. 6):

Рис. 6. Изобары на pV и pT-диаграммах

к оглавлению ▴

Изохорный процесс

Изохорный процесс, напомним, — это процесс, проходящий при постоянном объёме. При изохорном процессе меняются только давление газа и его температура.

Изохорный процесс представить себе очень просто: это процесс, идущий в жёстком сосуде фиксированного объёма (или в цилиндре под поршнем, когда поршень закреплён).

Пусть идеальный газ совершает изохорный процесс в сосуде объёмом V. Опять-таки рассмотрим два произвольных состояния газа с параметрами p_1, V, T_1 и p_2, V, T_2. Имеем:

p_1V=frac{displaystyle m}{displaystyle mu vphantom{1^a}}RT_1,

p_2V=frac{displaystyle m}{displaystyle mu vphantom{1^a}}RT_2.

Делим эти уравнения друг на друга:

frac{displaystyle p_1}{displaystyle p_2 vphantom{1^a}}=frac{displaystyle T_1}{displaystyle T_2 vphantom{1^a}}.

Как и при выводе закона Гей-Люссака, «разносим» индексы в разные части:

frac{displaystyle p_1}{displaystyle T_1 vphantom{1^a}}=frac{displaystyle p_2}{displaystyle T_2 vphantom{1^a}}. (7)

Ввиду произвольности выбора состояний мы приходим к закону Шарля:

frac{displaystyle p}{displaystyle T vphantom{1^a}}=const. (8)

Иными словами, при постоянном объёме газа его давление прямо пропорционально температуре:

p=const cdot T. (9)

Увеличение давления газа фиксированного объёма при его нагревании — вещь совершенно очевидная с физической точки зрения. Вы сами легко это объясните.

к оглавлению ▴

Графики изохорного процесса

График изохорного процесса называется изохорой. На pT-диаграмме изохора p = const cdot T является прямой линией (рис. 7):

Рис. 7. Изохора на pT-диаграмме

Смысл пунктирного участка тот же: неадекватность модели идеального газа при низких температурах.

Далее, чем больше объём, тем ниже идёт изохора на pTдиаграмме (рис. 8):

Рис. 8. Чем ниже изохора, тем больше объём

Доказательство аналогично предыдущему. Фиксируем температуру T и видим, что p_2 < p_1. Но при фиксированной температуре давление тем меньше, чем больше объём (снова закон Бойля — Мариотта). Стало быть, V_2 > V_1.

В оставшихся двух системах координат изохора является прямой линией, перпендикулярной оси V(рис. 9):

Рис. 9. Изохоры на pV и VT-диаграммах

Законы Бойля — Мариотта, Гей-Люссака и Шарля называются также газовыми законами.

Мы вывели газовые законы из уравнения Менделеева — Клапейрона. Но исторически всё было наоборот: газовые законы были установлены экспериментально, и намного раньше. Уравнение состояния появилось впоследствии как их обобщение.

Спасибо за то, что пользуйтесь нашими публикациями.
Информация на странице «Изопроцессы» подготовлена нашими редакторами специально, чтобы помочь вам в освоении предмета и подготовке к экзаменам.
Чтобы успешно сдать нужные и поступить в ВУЗ или техникум нужно использовать все инструменты: учеба, контрольные, олимпиады, онлайн-лекции, видеоуроки, сборники заданий.
Также вы можете воспользоваться другими материалами из данного раздела.

Публикация обновлена:
08.05.2023

ЛАБОРАТОРНАЯ РАБОТА 11

Изучение
зависимости давления газа
от его объёма

Цель
работы:
исследовать экспериментальный
газовый закон (изотермический) и установить связь между изменением давления и изменением
объёма газа.

Введение

Газ оказывает давление на стенки сосуда. Это давление обусловлено
столкновением молекул газа со стенками сосуда.

Уменьшение объёма, занимаемого газом (при неизменной массе газа),
приводит к тому, что в каждой единице объёма газа оказывается большее количество
молекул, число ударов молекул о стенки сосуда возрастает, и давление газа на
стенки сосуда увеличивается. Увеличение объёма (при постоянных массе и
температуре газа) приводит к уменьшению давления газа на стенки сосуда.

Впервые математическая зависимость давления газа от его объёма
PV = const была экспериментально получена английским ученым Р. Бойлем в 1662
г. и французским учёным Э. Мариоттом в 1676 г., поэтому она известна как закон
Бойля—Мариотта
.

Описание
эксперимента

В
этой работе вам предлагается экспериментально определить зависимость давления газа
в сосуде от объёма газа. Так как процесс изменения объёма происходит медленно и
температура газа при этом не успевает заметно измениться на протяжении всего
эксперимента, процесс можно считать изотермическим. Это возможно только в том
случае, если процесс изменения объёма будет происходить достаточно медленно.

В
качестве газа будет использоваться воздух, находящийся внутри шприца, подсоединённого
к датчику давления газа. При перемещении поршня шприца будет изменяться объём,
занимаемый воздухом, соответственно будет изменяться и давление воздуха.

Изменение
давления будет регистрироваться с помощью датчика давления газа.

Оборудование
и материалы

Наименование

Кол-во,
шт.

1

УИОД

1

2

ПО LabQuest App

1

3

Датчик давления
газа

1

4

Шприц объёмом 20
мл (входит в комплект датчика давления газа)

1

Оборудование
для лабораторной работы представлено на рис. 1.

Рис. 1

Задачи

1.   Экспериментально исследовать зависимость
давления газа от его объёма
при постоянной
температуре.

2.   Проанализировать полученные данные.

3.   Сделать
выводы.

Выполнение
работы

1.   Ознакомьтесь
с общими правилами техники безопасности при проведении практических работ в
кабинете физики.

2.   Подключите датчик
к УИОД. Выберите в меню Файл пункт Новый. Если все сделано правильно,
датчик определится автоматически, и на экране устройства вы увидите показания
датчика.

3.   Прежде чем подсоединить шприц к датчику давления газа, переместите
поршень шприца так, чтобы передняя часть внутреннего чёрного кольца поршня
остановилась у отметки 10 мл.

4.   Подсоедините шприц к клапану датчика давления газа как показано на
рисунке 2.

Рис. 2

5.   Измените
настройки измерения и обработки данных:

а – нажмите на поле Режим, нажмите
на вкладку Синхронизация и выберите в ниспадающем списке пункт События
с точкой входа;

б введите
в дополнительном окне в поле Имя название оси ХОбъём (
Objem),
в поле Единицы измерения – мл (
ml);

в – подтвердите выбор, нажав кнопку OK.

6.   Запустите
процесс измерения, нажав кнопку . Эксперимент удобнее выполнять вдвоём: один
ученик работает со шприцем, а другой – с компьютером.

7.   Переместите
поршень шприца так, чтобы передняя часть внутреннего чёрного кольца
остановилась у отметки 5 мл. Дождитесь, когда показания давления перестанут изменяться
. Нажмите кнопку KeepNew2, в поле Объём введите
цифру 5.
Подтвердите
выбор, нажав на кнопку ОК.

8.   Повторите
работу по пункту 7 несколько раз, увеличивая объём цилиндра шприца пошагово до
7,5; 10; 12,5; 15; 17,5 и 20 мл. В поле Объём вводите соответствующие
значения. Прилагайте усилие, чтобы удержать поршень шприца в нужном положении.

9.   Остановите
процесс измерений, нажав кнопку .

10. Занесите
в таблицу значения давления газа и его объёма для каждой точки графика,
округляя значение давления газа до 0,1 кПа. Для того чтобы выяснить значение
давления газа в любой точке графика, прикоснитесь к ней стилусом. В поле,
расположенном справа от графика, отобразится соответствующее числовое значение.

Также
вы можете перейти на вкладку Таблица и перенести оттуда полученные
данные давления газа в отчёт, округляя их до 0,1 кПа.

Проанализируйте
полученные данные.

11. При
наличии подключенного к УИОД принтера можете распечатать полученный результат.
Для этого в меню Файл выберите команду Печать, затем пункт График
и Таблица. Подпишите график и таблицу.

12. Ответьте
на контрольные вопросы.

13. Сделайте
вывод и заполните бланк отчёта.

Контрольные
вопросы

1.   Как объяснить давление газа на стенки сосуда?

2.   Как
изменится давление газа при увеличении его объёма в два раза, если масса газа и его температура не изменяются?
Сравните полученные в эксперименте значения давления газа при значениях объёмов,
равных 5 и 10 мл.

3.   Что
можно сказать о зависимости давления газа от занимаемого газом объёма:
зависимость прямо пропорциональная или обратно пропорциональная?

4.   На основании полученных данных сделайте
предположение о том, каким будет давление, если объём цилиндра шприца увеличить
до 40 мл? Поясните свой ответ.

5.   На основании
полученных данных сделайте предположение о том, каким будет давление газа, если
объём цилиндра шприца уменьшить до 2,5 мл? Поясните свой ответ.

6.   Какие факторы влияли на точность эксперимента?
Что нужно сделать, чтобы повысить точность эксперимента?

7.   Как можно объяснить увеличение давления газа
при умень­шении его объёма, если масса и температура газа не изменяются?

Дополнительные
задания (используйте
ПО Logger Pro)

1.   Постройте график зависимости давления (p)
газа от величины, обратной объему (
1/V).

2.   Определите
количество вещества — воздуха (в моль) в поршне. Используйте для расчёта закон
Менделеева—Клапейрона,

3.   Определите
массу воздуха в шприце, используя формулу , где М
= 29
10–3 кг/моль — средняя молярная масса
воздуха. Значение ν вы определили в предыдущем задании.

Понравилась статья? Поделить с друзьями:
  • Как найти драйвер для камеры ноутбука
  • Как составить поучение одноклассникам
  • Как найти украденный телефон через интернет самостоятельно
  • Как найти средний коэффициент мощности
  • Как исправить черты характера симс 4